Вопросы с подвохом с ответами для девушек: Страница не найдена | Блог Семья и дети

Содержание

Загадки на логику с подвохом | Онлайн-школа для детей — занятия для детей онлайн

Решение различных задач и загадок, способствующих развитию логического мышления, само по себе сложное и затягивающее в «омут знаний» занятие. А когда задачки еще и с подвохом, думать тяжелее вдвойне: ответ на вопрос может оказаться предельно простым и ввести в «ступор» даже бывалого математика.

Не нервничайте раньше времени и подключите к решению загадок детей, а еще лучше – зайдите с ними на сайт Умназии и составьте индивидуальную программу. Развивайте пять навыков мышления вместе с нами и будьте уверены в том, что ваш ребенок справится с любой логической (и даже алогической) школьной задачей.

Загадки с подвохом. В чем суть?

Вопросы с «подковыркой» содержат «тайну», которую нужно разгадать и детям, и взрослым. Здесь нет очевидных ответов, «прозрачных» решений и скучных алгоритмов. Верный ответ порой переворачивает сознание с ног на голову и заставляет взглянуть на мир по-другому. Дети учатся мыслить нестандартно и расширяют кругозор, взрослые смеются и (или) хватаются за голову в бесконечных попытках найти правильное решение.

Зачем это нужно?

Когда ребенок решает обычную задачу, он ищет самый «логичный» ответ на нее. Учится думать и анализировать, обрабатывать информацию, оценивать факты и делать выводы. При решении загадок с подвохом ему приходится менять алгоритм и подключать к работе не только левое «аналитическое» полушарие, но и «творческое» правое, которое оперирует вдохновением и фантазией. Найти правильный ответ помогает и то, и другое: в загадках с подвохом важно не только соотнести аргументы, посылки и выводы, но и предугадать неоднозначность самой отгадки.

Загадки с подвохом для детей

Начнем с малого и решим 5 загадок для самых маленьких. Взрослым ответ покажется очевидным, а вот ребятам придется подумать (но может быть и наоборот). Чур, не подсказывать!

Загадка №1
Что ты никогда не сможешь съесть на завтрак?

Показать ответ

Ответ: Обед и/или ужин.

Загадка №2.
Что может путешествовать по свету, оставаясь в одном и том же углу?

Показать ответ

Ответ: Почтовая марка.

Загадка №3
В одноэтажном розовом доме жил розовый человек, розовый кот, розовая рыбка, был розовый компьютер, розовое кресло, розовый стол, розовый телефон, розовая душевая кабина – все было розовым! Какого цвета была лестница?

Показать ответ

Ответ: В этом доме не было лестницы, потому что это был одноэтажный дом.

Загадка №4
Что настолько же огромное, как слон, но ничего не весит?

Показать ответ

Ответ: Тень слона.

Загадка №5
Какое слово в словаре написано неправильно?

Показать ответ

Ответ: Это слово «неправильно».

Мы знаем, что вам и вашим детям, как минимум, было интересно. А как максимум – вы серьезно задумались и хотите идти дальше!

СМЕШНЫЕ ЗАГАДКИ НА ЛОГИКУ

Подойдут для всех возрастных категорий и потребуют максимально нестандартных решений. Не заставят скучать, рассмешат и научат мыслить логически даже самого нелогичного человека. Переходим к изучению и поиску правильного ответа!

Загадка №1
У каких волос есть сходство с океаном?

Показать ответ

Ответ: У волнистых.

Загадка №2
Мужчина гулял в парке, когда на улице пошел дождь. У мужчины не было зонтика, и он не мог накинуть на себя плащ или накрыться шляпой. На нем не осталось сухой одежды, однако ни один волос на его голове не промок. Как это могло произойти?

Показать ответ

Ответ: Мужчина был лысый.

Загадка №3
Какой рукой удобнее размешивать сахар в стакане с чаем?

Показать ответ

Ответ: Рукой, которая держит ложку.

Загадка №4
На что больше всего похожа половинка яблока?

Показать ответ

Ответ: На вторую половинку этого же яблока.

Загадка №5
Какое изобретение позволяет смотреть сквозь стены?

Показать ответ

Ответ: Ничего сверхъестественного. Сквозь стены мы смотрим через окно.

Посмеялись? Если да, мы вас поздравляем: ученые утверждают, что смех продлевает жизнь. Если нет, не грустите – попробуйте еще раз вчитаться в условие и соотнести его с правильным ответом. Даже если после этого на вашем лице нет и намека на улыбку, будьте уверены, вы старались не зря: развитое логическое мышление непременно пригодится в повседневной жизни вам и вашему ребенку.

Трудные загадки с подвохом

Нет, мы еще не закончили. Последние пять загадок будут особенно сложными: здесь детям понадобится помощь взрослых. Опять придется мобилизовать оба полушария мозга, немного понервничать и вспомнить кое-что из школьного курса обществознания. Поразмышляйте над ответом, «пораскиньте» мозгами и найдите то самое решение!

Загадка №1
Человек выпрыгнул из самолёта без парашюта. Он приземлился на твёрдый грунт, но остался невредимым. Почему?

Показать ответ

Ответ: Самолёт стоял на земле.

Загадка №2
«Мой брат – адвокат», — говорит бухгалтер. Адвокат говорит, что у него нет брата. Кто из них лжет?

Показать ответ

Ответ: Никто, потому что бухгалтер – сестра адвоката.

Загадка №3
Мужчина ехал по дороге на своем грузовике. Фары не были включены. Луна не светила. Перед грузовиком женщина в черной одежде переходила дорогу. Догадайтесь, как мужчина ее увидел.

Показать ответ

Ответ: Женщину было отчетливо видно, потому что это происходило днем, а не ночью.

Загадка №4
Однажды за завтраком девушка уронила кольцо в чашку с кофе. Однако кольцо осталось сухим. Почему?

Показать ответ

Ответ: Кофе не был напитком: он был в зернах. Если вы ответили, что кофе был молотый или растворимый, вы тоже правы.

Загадка №5
Что все время приходит, но никак не придет?

Показать ответ

Ответ: Завтра.

Ура! Мы с вами поработали на отлично и стали чуть-чуть умнее.


Хотите еще?
Приводите ребенка в Умназию и развивайтесь вместе с нами.

В вашем распоряжении более 4000 авторских задач, выбор траектории обучения, игровой формат и формирование каждого из пяти навыков. Логика, эрудиция, внимание, память и обучаемость – настраивайте и выбирайте наиболее интересные для ребенка задания, получайте награды и вместе радуйтесь новым победам.
Начните заниматься в Умназии уже сегодня!

Математика и логика для детей 7-13 лет

Развиваем логическое мышление через решение сюжетных математических задач в интерактивном игровом формате

узнать подробнее


Читайте также:


 

Логические загадки с подвохом для детей и для взрослых


Загадки с подвохом для взрослых


Перед вами серьезные и забавные загадки на логику и смекалку.

Внимательно читайте или слушайте вопросы и не стесняйтесь придумывать самые нестандартные
ответы.


В некоторых хитрых загадках ответ лежит на поверхности. Чтобы отгадать другие нужно как следует
«пошевелить извилинами». Отличная подборка для праздника: и мозги
«разомнете», и друзей развеселите.


К реке подходят 2 человека. У берега стоит лодка, она может выдержать только
одного.

Оба человека смогли переправиться на другой берег.

Как им это удалось?


Что будет, если повернуть направо три раза?


Как найти площадь Ленина?


Какое самое длинное слово на русском языке?


Под каким деревом сидит заяц, когда идет дождь?


На столе лежат линейка, карандаш, циркуль и резинка. На листе бумаги нужно
начертить окружность. С чего Вы начнете?


Как называются серёжки для простаков?


В каком случае шесть детей, две собаки, четверо взрослых, забравшись под
один зонтик, не намокнут?


Сколько яиц можно съесть натощак?

Загадки с подвохом

Загадки с подвохом — это загадки на внимание, сообразительность и чувство юмора. Их нужно задавать быстро, не давая много времени на раздумья.

 

Готовые сценарии для проведения квеста. Подробную информацию можно посмотреть, нажав на интересующую картинку.

 

Весёлые загадки на смекалку с ответами

На любом празднике удачно подобранные загадки будут хорошим развлечением для гостей и обязательно поднимут настроение всем без исключения! С помощью данных загадок вы можете:

  • Использовать их для заполнения пауз между играми, конкурсами и танцами.
  • Устроить весёлую застольную викторину. Для того, чтобы заинтересовать гостей и облегчить процесс определения победителей, можно купить побольше денег от «банка приколов» и вручать их за правильные ответы. Присутствующие мгновенно включаются в игру и будут стараться заработать много «денежек». Вместо «денег» можно приготовить узкие цветные полоски бумаги и, например, нарисовать на них восклицательный знак – за правильный ответ. В конце викторины желательно наградить сувенирами (или шуточными медалями) трёх самых смекалистых игроков, т.е. тех, у кого оказалось больше всего «денег» или цветных бумажек.
  • Устроить командное соревнование (особенно оно подойдет для подростков): разбить игроков на несколько команд, каждой команде выдать листы с одинаковыми загадками и ручки. Чья команда даст больше верных ответов за отведенное ведущим время, та и будет победителем.

Сколько на берёзе яблок, если у нее восемь веток, на каждой из них по 5 яблок?

Показать

(На берёзе не растут яблоки)

[свернуть]

Где находятся города без домов, реки без воды, а леса без деревьев?

Показать

(На географической карте)

[свернуть]

Несла старушка на базар 50 яиц, а дно упало. Сколько яиц осталось? Ведущий произносит «а дно» как «одно»

Показать

(Все разбились, так как упало дно)

[свернуть]

Сидели на дереве шесть глухарей. Пришел охотник и убил одного глухаря. Сколько птиц осталось?

Показать

(Ни одной — остальные улетели)

[свернуть]

Как спрыгнуть с двадцатиметровой лестницы и не разбиться?

Показать

(Спрыгнуть с первой ступеньки, или, для смелых и ловких, — со второй или третьей)

[свернуть]

Шёл Иван в Москву, зашёл на мельницу. Там 4 окошка, на каждом окошке по 4 кошки. У каждой кошки по 4 котенка, а у каждого котёнка по 4 мышки. Сколько всего ног?

Показать

(Две ноги — у Ивана, остальные — лапы)

[свернуть]

Какой рукой лучше размешивать чай?

Показать

(Чай лучше размешивать ложкой)

[свернуть]

Сколько горошин может войти в один стакан?

Показать

(Нисколько – горошины не ходят)

[свернуть]

Перед кем все люди без исключения снимают шляпы?

Показать

(Перед парикмахером)

[свернуть]

У кого есть шапка без головы и нога без сапога?

Показать

(У гриба)

[свернуть]

За что обычно учеников выгоняют из класса?

Показать

(За дверь)

[свернуть]

Что за обедом нужнее всего?

Показать

(Рот)

[свернуть]

Что нельзя съесть на завтрак?

Показать

(Ужин и обед)

[свернуть]

Какими нотами можно измерить пространство?

Показать

(Ми – ля – ми)

[свернуть]

Что можно приготовить, но нельзя съесть?

Показать

(Уроки)

[свернуть]

Что делает сторож, когда у него на шапке сидит воробей?

Показать

(Спит)

[свернуть]

Кто под проливным дождем не намочит волосы?

Показать

(Лысый)

[свернуть]

Что может быть больше слона и одновременно невесомым?

Показать

(Тень слона)

[свернуть]

На какое дерево садится ворона во время дождя?

Показать

(На мокрое)

[свернуть]

Что станет больше, если его поставить вверх ногами?

Показать

(Число 6)

[свернуть]

Каких камней в море не встретишь?

Показать

(Сухих)

[свернуть]

Что станет с красным шёлковым платком, если опустить его на 5 минут на дно моря?

Показать

(Будет мокрым)

[свернуть]

Как сорвать ветку, чтоб не вспугнуть птицу?

Показать

(Надо подождать, когда она улетит)

[свернуть]

Из какой посуды нельзя ничего поесть?

Показать

(Из пустой)

[свернуть]

Чем заканчивается день и ночь?

Показать

(Мягким знаком)

[свернуть]

Когда человек бывает в комнате без головы?

Показать

(Когда высовывает её из окна)

[свернуть]

Может ли страус назвать себя птицей?

Показать

(Нет, страус не умеет говорить)

[свернуть]

Что у человека под ногами, когда он идет по мосту?

Показать

(Подошва обуви)

[свернуть]

Какой узел нельзя развязать?

Показать

(Железнодорожный)

[свернуть]

Можно ли в решете воды принести?

Показать

(Можно, если она там замерзнет)

[свернуть]

Сколько месяцев в году имеют 28 дней?

Показать

(Все)

[свернуть]

Какая разница между слоном и блохой?

Показать

(У слона могут быть блохи, а у блохи не может быть слонов)

[свернуть]

За чем вода в бутылке?

Показать

(За стеклом)

[свернуть]

Что может путешествовать по миру, оставаясь в одном и том же углу?

Показать

(Почтовая марка)

[свернуть]

Что все люди на земле делают одновременно?

Показать

(Становятся старше)

[свернуть]

Серёжки для простаков?

Показать

(Лапша)

[свернуть]

Когда козе исполнится семь лет, что будет дальше?

Показать

(Пойдёт восьмой год)

[свернуть]

Каким гребнем нельзя расчесывать голову?

Показать

(Петушиным)

[свернуть]

Когда мальчика называют женским именем?

Показать

(Когда он долго спит – Соня)

[свернуть]

Как написать слово «мышеловка» пятью буквами?

Показать

(Кошка)

[свернуть]

Что идет, оставаясь на месте?

Показать

(Дорога)

[свернуть]

Когда человек может мчаться со скоростью гоночного автомобиля?

Показать

(Когда он сидит в нем)

[свернуть]

Какие часы показывают верное время только два раза в сутки?

Показать

(Те, которые остановились)

[свернуть]

Что можно увидеть с закрытыми глазами?

Показать

(Сон)

[свернуть]

Рекомендуем посмотреть:

За чем мы едим?

Показать

(За столом)

[свернуть]

Что легче: килограмм ваты или килограмм железа?

Показать

(Одинаково — по килограмму)

[свернуть]

Какой конь не ест овса?

Показать

(Шахматный, спортивный)

[свернуть]

Какое колесо не крутится при правом развороте?

Показать

(Запасное)

[свернуть]

По чему, когда захочешь спать, идешь на кровать?

Показать

(По полу)

[свернуть]

Какое животное может легко поменять голову?

Показать

(Вошь)

[свернуть]

Когда черной кошке легче всего пробраться в дом?

Показать

(Когда дверь открыта)

[свернуть]

На окне сидит, по-французски говорит?

Показать

(Француз)

[свернуть]

Какая разница между молнией и электричеством?

Показать

(За молнию не надо платить)

[свернуть]

Какой возраст у ребёнка самый хороший?

Показать

(Когда вы уже не водите его за руку, а он ещё не водит вас за нос)

[свернуть]

Что теплее шубы?

Показать

(Две шубы)

[свернуть]

Две спины, одна голова, шесть ног. Что это такое?

Показать

(Человек на стуле)

[свернуть]

Что это: две руки, два крыла, два хвоста, три головы, три туловища и восемь ног?

Показать

(Всадник, держащий в руках курицу)

[свернуть]

Маленькая, сморщенная, есть в каждой женщине?

Показать

(Изюминка)

[свернуть]

Назовите слово, в котором 40 гласных

Показать

(Сорока — сорок «А»)

[свернуть]

Рта нет, языка нет, а говорит о каждом

Показать

(Зеркало)

[свернуть]

На какой вопрос нельзя ответить «да»?

Показать

(Вы спите?)

[свернуть]

На какой вопрос нельзя ответить «нет»?

Показать

(Вы живы?)

[свернуть]

В каком случае шесть детей, две собаки, четверо взрослых, забравшись под один зонтик, не намокнут?

Показать

(Если не будет дождя)

[свернуть]

Что есть только у слонов, и ни у каких других животных больше нет?

Показать

(Слонята)

[свернуть]

Что такое 90-60-90?

Показать

(Езда мимо гаишника)

[свернуть]

Маленький, желтенький, в земле ковыряется

Показать

(Вьетнамец мину ищет)

[свернуть]

Маленький, желтенький, в небе кувыркается

Показать

(Вьетнамец мину нашел)

[свернуть]

Два гвоздя упали в воду. Как фамилия грузина?

Показать

(Заржавели)

[свернуть]

Что такое: самое доброе в мире привидение с моторчиком?

Показать

(Запорожец)

[свернуть]

В небе одна, в земле нету, а у бабы их целых две

Показать

(Буква «Б»)

[свернуть]

Что такое: зелёное, лысое и скачет?

Показать

(Солдат на дискотеке)

[свернуть]

Шёл охотник мимо башни с часами. Достал ружьё и выстрелил. Куда он попал?

Показать

(В полицию)

[свернуть]

Без окон, без дверей, а внутри сидит еврей? Что это?

Показать

(Сара беременна)

[свернуть]

Что это такое: на потолке сидит, лампочку грызет?

Показать

(Потолковый лампогрыз)

[свернуть]

Маленький, серенький, на слона похож

Показать

(Слоненок)

[свернуть]

Чем их больше, тем вес меньше. Что это?

Показать

(Дырки)

[свернуть]

Что принадлежит вам, однако другие этим пользуются чаще, чем вы сами?

Показать

(Имя)

[свернуть]

У человека — одна, у вороны — две, у медведя — ни одной. Что это?

Показать

(Буква «О»)

[свернуть]

Сколько яиц можно съесть натощак?

Показать

(Одно)

[свернуть]

Сколько минут нужно варить крутое яйцо?

Показать

(Нисколько — оно уже сварено)

[свернуть]

Что нужно сделать, если вы сели в машину, а ноги до педалей не достают?

Показать

(Пересесть на водительское сиденье)

[свернуть]

Растение, которое всё знает.

Показать

(Хрен)

[свернуть]

Как называется тень от растения хрен?

Показать

(Хренотень)

[свернуть]

Воробей может съесть горсточку зерна, а лошадь не может. Почему?

Показать

(Воробей слишком маленький, чтобы съесть лошадь)

[свернуть]

Светит, но не греет.

Показать

(15 лет строгого режима)

[свернуть]

Где ещё кроме ёлки дети кричат «Дед Мороз, выходи!»?

Показать

(Возле туалета)

[свернуть]

Почему у козы глаза грустные?

Показать

(Потому что муж козёл)

[свернуть]

Как поймать тигра в клетку?

Показать

(Тигра в клетку не бывает, тигр в полоску)

[свернуть]

Два слова, разбивающие все доказательства?

Показать

(И чё?)

[свернуть]

Когда же ребёнок станет конем?

Показать

(Никогда — он ребёнок)

[свернуть]

Как называют корову, которая не даёт молока?

Показать

(Жадина-говядина)

[свернуть]

Кто ещё кроме вампира не отображается в зеркале и не отбрасывает тень?

Показать

(Профессиональный фотограф)

[свернуть]

Самый дорогой кофе в мире?

Показать

(Кофе, пролитый на клавиатуру ноутбука)

[свернуть]

Рекомендуем посмотреть:

 

Прикольные загадки для уже разгулявшейся и не очень трезвой компании взрослых:

То висячий, то стоячий, то холодный, то горячий

Волосатая головка за щеку летает ловко

Показать

Зубная щётка

[свернуть]

Без рук, без ног, на бабу скок!

Показать

Коромысло

[свернуть]

Беру двумя руками, сую между ногами, пять минут потею, а потом балдею

Показать

Велотренажер

[свернуть]

В тёмной комнате, на белой простыне — два часа удовольствия

Показать

Киносеанс

[свернуть]

Волос на волос, тело на тело, и получается тёмное дело

Показать

Глаза

[свернуть]

Туда, сюда, обратно, тебе и мне приятно

Показать

Качели

[свернуть]

Красная головка, работает ловко

Показать

Дятел

[свернуть]

Между ног болтается, на «Х» начинается?

Показать

Хвост

[свернуть]

Мы ребята удалые — ищем щели половые

Показать

Тараканы

[свернуть]

Тихо сзади подошёл, дважды всунул и пошёл

Показать

Тапочки

[свернуть]

Стоит баба на полу, приоткрыв свою дыру

Показать

Печка

[свернуть]

Ты помни его немножко — станет твёрдый, как картошка

Показать

Снежок

[свернуть]

У какого молодца утром капает с конца?

Показать

Самовар или Кран

[свернуть]

Мальчик с девочкой в траве что-то делали на «Е»

Показать

ели землянику

[свернуть]

Чтобы спереди погладить, надо сзади полизать

Показать

Почтовая марка

[свернуть]

Что такое: входит сухим, выходит влажным, дарит тепло и радость?

Показать

Пакетик с чаем

[свернуть]

Назовите слово из 5 букв, содержащее буквы: п, з, д, а.

Показать

Запад

[свернуть]

Выше колена, пониже пупка, дырка такая, что влезет рука

Показать

Карман

[свернуть]

Что такое синее золото?

Показать

Любимая жена напилась

[свернуть]

Он от бабушки ушёл, и от дедушки ушёл… Что это?

Показать

Секс

[свернуть]

Почему у Кощея Бессмертного не было детей?

Показать

Потому что у него всего одно яйцо, и то за тридевять земель

[свернуть]

Из трех букв состоит,

На «Х» начинается,

Когда работает — стоит,

Когда кончает — кланяется.

Что такое: у мужчины на 3 буквы, у женщины на 5 букв?

Показать

Чуб, чёлка

[свернуть]

Что удлиняется, когда его берут в руки, пропускают между грудей и засовывают в отверстие?

Показать

Ремень безопасности

[свернуть]

Также вас могут заинтересовать:

  • Квест для мужчины: для мужа, для любимого человека, для друга на День рождения или Юбилей
  • Квест для женщины: для жены, для любимой девушки, для подруги, сестры, дочки или сотрудницы на День рождения (Юбилей)
  • Командная квест-игра в помещении (в квартире, коттедже, офисе), или поиск спрятанного сюрприза по подсказкам
  • Квест-игра, или поиск спрятанных подарков на День рождения ребёнка 6, 7, 8, 9, 10 лет
  • Командная квест-игра для детей
  • Новогодняя квест-игра: поиск спрятанных подарков в помещении (в квартире, коттедже, офисе)

Вопросы для девушек с подвохом. ТОП-4 | Реальные Факты

Каждый из нас сталкивался с этой ситуацией — легко общаешься с девушкой,и вроде общение развивается…

Однако есть моменты, когда стоит задать девушке каверзные вопросы, если хочешь сделать определенные выводы из ее ответов. Зачем вообще задавать такие вопросы с подвохом? А затем, что по сделанным выводам можно узнать больше о девушке, ее характере, предпочтениям и многом другом. В обычной ситуации, без твоей инициативы, она может и не показывать своих скрытых сторон.

Если ты действительно хочешь хороших, качественных отношений с девушкой, то стоит говорить не только о ее любимых телешоу.

Какие каверзные вопросы с подвохом можно задать девушке для того, чтобы лучше ее узнать и понять, что она любит, что ненавидит, и что она вообще за человек?

Спрашивать ее о том, где работает или сколько осталось учиться до выпуска не заставит биться ее сердце чаще. Кстати, такие вопросы вообще не стоит задавать на стадии соблазнения.
Извини, конечно, но если ты будешь спрашивать девушку о разных скучных вещах – это не заставит ее думать про темя, когда она будет удовлетворяться .

Задавай ей вопросы, которые вызывают сильные эмоции. Дай ей понять, что тебе интересно ее мнение. Помоги ей почувствовать себя особенной. Общайся с девушкой так, чтобы узнать ее с необычной стороны.

Если ты сделаешь это, она подумает:
«Он невероятный! Я еще не встречала парня, который меня о таком спрашивал. Он первый молодой человек, который дает мен ощущение заботы обо мне. Я не знаю, как такое возможно, но у меня сильное влечение к нему.»
Давай разберем основные каверзные вопросы:

1.Если могла бы выбрать любое место, где ты хотела бы сейчас оказаться?

Это удивительный и глубокий вопрос, который можно задать девушке, которая нравится. Если ты действительно ей нравишься, она скажет, что хочет быть здесь, рядом с тобой, и нигде больше.
Если же она действительно ответит, что хочет быть здесь и сейчас рядом с тобой, поменяй слегка вопрос. Спроси, в какой стране и обстановке хотела бы оказаться.
Это может многое рассказать о ее личности.
Девушка, которая говорит о желании оказаться в дорогом бутике в центре Лондона очень сильно отличается от дамы, которая хочет в деревянный дом в лесу с камином. Этот вопрос раскрывает желание о месте и стиле жизни.

2.Чего ты боишься больше всего?

Вопрос о страхах также помогает лучше узнать человека. Он также может показать, имеешь ли ты дело с идиоткой или девушкой, которая имеет все же больше мозгов, чем Ольга Бузова.
«Я так боюсь, что однажды мое любимое телешоу Дом 2 закроется. Я тогда просто буквально повешусь!»
«Я думаю, самый сильный страх – это когда мой родной человек страдает, и я не могу ему помочь».Я надеюсь, ты видишь разницу между ответами. И я надеюсь, ты тут же свалишь от девушки, которая так любит Дом 2.

3.Какая у тебя самая сильная страсть?

Я знаю, что это звучит как клише. Сегодня каждый говорит о целях в жизни и увлечениях. Но все-таки это хороший вопрос.
Чтобы отношения были гармоничными, у тебя с девушкой должны быть общие увлечения.
Я знаю о поговорке, что противоположности притягиваются. И нет ничего страшного, что вы будете немного отличаться. Но из собственного опыта скажу, что общение девушки, которая любит шопиться днями напролет и парня, ведущего аскетичный образ жизни, долго не продлится.
Узнай о ее самых сильных желаниях в жизни, и ты поймешь, подходит она тебе или нет.

4.Как часто ты видишься со своими лучшими друзьями?

Дружба очень важна для меня. Конечно, я не могу регулярно и часто встречаться с друзьями в силу работы и частых поездок. Однако, я всегда нахожусь в контакте со своими друзьями и когда выдается свободное время, стараюсь увидеться с ними.
Если дружба важна для тебя, убедись, что твоя новая подруга имеет такое же отношение к ней.
Зачем тебе тратить время на общение с девушкой, которой до фонаря её друзья, либо которых у нее нет вовсе по вышеприведенной причине?

Девушка, которой наплевать на дружбу, также будет общаться и с остальными людьми, появляющимися в ее жизни. В таком случае, стоит найти другую девушку, для которой слово «дружба» имеет значение.

Загадки. Прикольные и смешные загадки с подвохом:

Размер

Толщина

Фон

 Скрыть 
 ответы 
 +шрифт 

 —шрифт 
 +жирн 

 —жирн 

White
Cyan
LGreen
GYellw
DpSkBl
Coral
DPink1
DPink2
SkBlue
Orange
OlivD1
OlivD2
LBlue
PGreen
Yellow
Gold
Blue
Green
Wheat
Chocol
Salmon
Red
HPink
DPink
Pink
VioRed
Magent
Violet
Plum
Purple
OrRed
Bisque
Bisqu2
LemC1
LemC2
Corns2
Honey2
Turqu1
Turqu2
SGrn1
SGrn2
Orchi1
Orang1
Gray
golrod



Что такое диетическое яйцо?

Ответ

Это яйцо, снесенное курицей, находящейся на диете.



О чем думал Мюнхгаузен, когда летел на ядре?

Ответ

Ничего себе в кегельбан поиграл.



Кто получит бесплатный сыр в мышеловке?

Ответ

Вторая мышка.



Что такое глобус?

Ответ

Это чучело земли.



Какая самая не любимая фраза сапера?

Ответ

Одна нога здесь, другая – там.



Что такое МАРС?

Ответ

Это собачий холод и толстый-толстый слой космической пыли.



Что такое чистоплотность?

Ответ

Это чистомасса на чистообъем.



Куда каждый раз падает ядерная бомба?

Ответ

В эпицентр.



Кто такой истинный патриот своей страны?

Ответ

Это тот, кто больше нигде не был.



Как вы думаете, что это такое : черно-белое на 16 колесах.

Ответ

Это зебра на роликовых коньках.


Поделитесь с друзьями:


Как вы думаете, в чем разница между футболистом и уличным пешеходом?

Ответ

На красный пешеход стоит, а футболист уходит.



Если работа не волк, то что тогда?

Ответ

Произведение силы на расстояние.



Что такое – не пойман, не вор?

Ответ

Депутатская неприкосновенность.



Кто такие вертолёты?

Ответ

Это души погибших танков.



Как называется бумеранг, который не вернулся?

Ответ

Палка.



Если оптимист изобретает колесо, то что изобретает пессимист?

Ответ

Запасное колесо.



Какие зубы у человека появляются последними?

Ответ

Искусственные.



Если во время соревнований сходит лавина, то что происходит с лыжниками?

Ответ

Все финишируют одновременно.



Кто в армии служил, тот в цирке …?

Ответ

не смеется.



Закончите сказку : «3 дня и 3 ночи скакал Иван Царевич, пока …»?

Ответ

пока скакалку не отобрали.


Поделитесь с друзьями:

Вопросы-головоломки на собеседованиях с ответами. Часть 1 — Work.ua

Логические задачи кандидатам на вакансию задают, чтобы оценить их аналитические способности. С подобными вопросами может столкнуться каждый. Но даже, если с вами это не случится — изучить их для подготовки будет просто интересно.

Соискателям, столкнувшимся с головоломками в процессе собеседования, не стоит паниковать и теряться. Как правило, в большинстве случаев достаточно показать ход ваших мыслей и пути решения поставленной задачи. Вполне может быть, что у задачи и нет правильного решения или однозначного ответа. Ни в коем случае нельзя говорить «я не знаю» или «затрудняюсь ответить», просто размышляйте вслух.

Чтобы вы знали к чему быть готовыми и не растерялись, Work.ua собрал самые популярные вопросы-головоломки и подготовил ответы и рекомендации к ним. Кто знает, может вам попадутся именно эти задачи.

Все эти головоломки могут вам задать при собеседовании на вакансию программиста php.

Канализационные люки

Это, пожалуй, самый распространенный вопрос среди всех головоломок.

Вопрос: Почему канализационные люки круглые?

Ответ: Есть несколько вариантов. Так как диаметр круга одинаков, как его не крути, то круглый люк не может провалиться в колодец. У квадрата же, например, диагональ больше его сторон, поэтому крышка могла бы упасть. Также можно ответить, что круглые люки проще транспортировать и передвигать.

Лампочки

Вопрос: В закрытой комнате есть 3 лампочки, а в коридоре 3 выключателя. За какое минимальное открытие дверей можно определить какой выключатель к какой лампочке относится?

Ответ: За одно открытие. Включаем одновременно 2 выключателя, через некоторое время выключаем второй. Заходим в комнату: одна из лампочек осталась гореть — это первый выключатель, далее щупаем оставшиеся лампочки — теплая и будет вторым выключателем, а холодная соответственно третьим.

Торт

Вопрос: Как разделить торт на 8 равных частей тремя разрезами?

Ответ: Сперва нужно сделать 2 разреза крест на крест, поделив торт на 4 равных части. А затем разрезать торт горизонтально пополам. Ну и что, что куски стали невысокие, зато у вас 8 равных частей. Еще можно после первых двух разрезов сложить кусочки друг на друга и переполовинить одним разрезом.

Мертвый человек и спичка

Вопрос: Среди поля найден мертвый человек со спичкой в руках, следов нет. От чего он умер и при каких обстоятельствах?

Ответ: Человек умер от падения с самолета, который начал терять высоту, и авария была неизбежна. На всех пассажиров не хватило одного парашюта и они тянули жребий. Ему досталась короткая спичка, и он был вынужден прыгать без парашюта.

Автобус и мячи

Вопрос: Сколько теннисных мячей поместится в автобус?

Ответ: Точного правильного ответа не знает и сам рекрутер, так как не уточняется, что за мячи и автобус — их размеры не известны, и никто досконально это не проверял. Поэтому тут важен ход ваших мыслей, вы можете только предположить. Назовите примерные длину, ширину и высоту автобуса, размеры одного мяча. Посчитайте объем автобуса и мяча — так вы узнаете сколько мячей поместится в пустой автобус. Уменьшите примерно это значение с учетом сидений и других деталей автобуса, сделайте поправку на то, что мячи не квадратные и дайте ответ. В этом случае процесс поиска ответа важнее самого ответа. Варианты вопроса могут быть разные: мячи — футбольные, вместо автобуса — комната и т.п.

Таблетки

Вопрос: Доктор выдал пациенту 4 таблетки двух видов — по 2 таблетки каждого, которые нельзя отличить по внешнему виду. Таблетки надо выпить за два приема: утром по одной таблетке каждого вида и так же вечером. Если нарушить дозировку или не принять таблетки, то пациент умрет. Так вышло, что таблетки перемешались. Как пройти курс лечения и выжить?

Ответ: Конечно, можно сказать, что лучше пойти к врачу и попросить еще, все таки вопрос жизни и смерти. Но это могут быть единственные таблетки на Земле, доктор может исчезнуть при загадочных обстоятельствах и т.д. Так что ответить все же придется. К тому же все достаточно просто: нужно разделить каждую таблетку на 2 части и выпить по половинке каждой таблетки утром и вечером.

В заключение

Помните, что прежде всего такие задачи и головоломки предназначены для того, чтобы проверить поведение кандидата в нестандартных ситуациях, оценить способность к размышлению, творческому и логическому подходу. Увы, нередки случаи, когда интервьюер не может правильно интерпретировать результаты таких вопросов, или вовсе не понимает их предназначения. Но даже в этой ситуации уверенное поведение и стремление прийти к ответу покажут вас с лучшей стороны и увеличат шансы получить должность.

Читайте также: Вопросы-головоломки на собеседованиях с ответами. Часть 2



Чтобы оставить комментарий, нужно войти.

Вопросы для парня с подвохом

Оригинальные вопросы парню

  • С кем бы ты разделил свою участь, находясь на необитаемом острове?
  • Как быстро ты бы потратил 1 млн. долларов?
  • Как считаешь, зачем черту нужны рога?

Кто по твоему мнению глава семьи – мужчина или женщина?

  • Сколько раз в день ты чистишь зубы?
  • Как думаешь, после смерти ты попадешь в рай или ад?
  • Как долго ты можешь смотреть на себя в зеркало?
  • Когда для тебя заканчивается рабочий день?
  • Что бы ты кричал, когда падал со скалы вниз?
  • Какую пищу ты любишь кушать руками, а не стандартными приборами?
  • Кривые гримасы у тебя всегда вызывают смех?
  • Какой тебе нравится телевизионный проект?
  • Твоя будущая жена должна быть такой же хозяйкой как мама?
  • Ты бы проехал в метро в комнатных тапочках?
  • Ты проламываешь первый лед на лужах, когда идешь мимо?
  • Ты боишься застрять один в лифте?
  • Ты попарно складываешь носки после стирки или все лежат в одной куче?

Необычные вопросы парню

  • Можешь вспомнить свои ощущения после первого поцелуя с девушкой?
  • Какая профессия тебе наиболее по душе?
  • Ты бы завел курортный роман на отдыхе, будучи женатым?

Ты хотел бы иметь сразу нескольких жен, как в восточных странах?

  • Для тебя приемлемо плакать из-за обиды, предательства или еще чего-то?
  • В кругу твоих знакомых есть ли люди с криминальным прошлым?
  • Ты всегда придешь на помощь к незнакомому человеку, если это потребуется?
  • Ты любитель ходить на рыбалку ранним утром?
  • Тебе было бы не страшно спать в палатке ночью в лесу?
  • Какая еда вызывает у тебя не аппетит, а отвращение?
  • Хотел бы попробовать себя в роли костюмированного аниматора, который развлекает прохожих на улицах города?
  • Ты веришь в гадания по руке?
  • На твой взгляд, почему мы видим сны?
  • Какое твое самое раннее воспоминание из детства?
  • Как часто ты дерешься?
  • Если бы ты стал на время невидимкой, то за кем бы подсматривал?
  • Ты завидуешь парням, имеющим супер способности?

Коварные вопросы для парней

  • Ты просыпался в неизвестном ранее месте?
  • У тебя есть шанс нарушить закон безнаказанно. Какое преступление ты бы совершил?
  • Ты бы смог поменять свое отчество?
  • Как долго ты бы смог общаться в переписке с девушкой, не видя ее внешности?
  • Как ты воспринимаешь «черный» юмор над собой?
  • Для тебя важно быть верующим человеком?
  • Ты всегда доверяешь тому, что тебе говорят, или перепроверяешь сказанное?

Ты способен любить двух девушек одновременно?

  • Насколько дорогие подарки ты даришь близким людям?
  • При встрече с девушкой, на что первое в ее внешности ты обращаешь внимание?
  • Ты смог бы отрубить топором голову курице?
  • Ты выбираешь красивых блондинок или умных?
  • Сколько раз ты сможешь отжаться на перекладине?
  • Ты бы хотел порыбачить с лодки у берега океана?
  • Как часто ты рассматриваешь в ночном небе звезды?

Неожиданные вопросы парню

  • Ты прислушиваешься к соседским разговорам за стенкой?
  • Тебя трудно вытащить рано утром из теплой постели?
  • Ты когда-нибудь доил корову?
  • Ты бы смог прожить месяц в пещере горы с продовольствием и светом, но в одиночестве?
  • Ты когда-нибудь представлял ситуацию, что мог бы очнуться от наркоза во время операции?
  • Ты бы хотел иметь два сердца?
  • У тебя хорошо развита интуиция?

Что для тебя ужаснее – быть глухим или слепым?

  • Как часто ты посещаешь туалет за день?
  • Ты веришь в жизненный закон бумеранга?
  • Тебе что больше нравится — бальные танцы или фигурное катание?
  • Ты бы хотел оказаться на борту космического корабля?
  • Ты сел бы за руль авто ночью, если бы не светили фары?
  • Ты пользуешься зубочистками или чем-то другим?
  • Когда последний раз ты был в здании почты и что там делал?
  • Ты хотел бы сделать себе пластическую коррекцию на лице?
  • При необходимости ты бы сдал донорскую кровь для близкого человека?
  • Тебе больше по душе зима или осень?

Неловкие вопросы парням

  • Ты чистишь грязь у себя под ногтями?
  • Какая у тебя ассоциация с красным цветом?
  • О чем ты думаешь, когда видишь на дороге гаишника?
  • Ты бы ходил зимой на работу в валенках?

Что для тебя более важно – быть богатым или здоровым?

  • Для тебя понедельник – это тяжелый день?
  • Каких именно животных ты бы хотел понимать?
  • Что для тебя самая вредная пища?
  • За каким процессом тебе интересно было бы смотреть?
  • Ты бы хотел иметь крылья?
  • Как часто ты общаешься «по душам» со своими близкими или друзьями?
  • Ты допускаешь разовую измену в любовных отношениях?
  • Ты всегда перезваниваешь, если на телефоне есть пропущенные звонки?
  • Ты рассматриваешь случайный секс с коллегой по работе?
  • Тебя будет задевать тот факт, что твоя девушка зарабатывает гораздо больше?
  • В какой стране ты бы хотел прожить последние дни?
  • С каким овощем ты себя можешь сравнить?

Комментарии:

114 Вопросы-трюки с ответами [Смешные вопросы-трюки]

Вы можете считать, что достаточно умен, чтобы вас не обманули. Однако вопросы с подвохом могут поставить в тупик любого. Здравый смысл и опыт не помогут вам ответить на эти каверзные вопросы. Несмотря на то, что некоторые ответы логичны или основаны на здравом смысле, ваша первая мысль довольно часто неверна. Некоторые вопросы с подвохом выглядят смешно, некоторые представляют собой игру слов, а некоторые требуют другого взгляда на вещи. Попробуйте свои силы в ответах на все наши хитрые вопросы и поделитесь ими с друзьями.

Сложные вопросы

Следующий список умных и сложных вопросов заставит вас и ваших друзей почесать затылки и задуматься о том, насколько вы умны. Их приятно использовать на вечеринке, с коллегами на работе или в классе. Учителям, вожатым и артистам понравится наш список сложных вопросов.

  1. Вопрос: Что всегда приходит, но никогда не приходит?

    Завтра

  2. Вопрос: Что можно сломать, но не удержать?

    Обещание

  3. Вопрос: Что живет, если его накормить, и умирает, если вы его напоите?

    Fire

  4. Вопрос: Каким словом вы бы описали человека, у которого не все пальцы на одной руке?

    Нормально, потому что люди обычно держат половину пальцев на одной руке.

  5. Вопрос: Что можно поймать, что не брошено?

    Холодный

  6. Вопрос: Что, если у вас есть, вы хотите поделиться со мной, а если поделитесь, у вас нет?

    Секрет

  7. Вопрос: Если самолет упадет на границе США и Канады, где хоронят выживших?

    Выжившие не похоронены.

  8. Вопрос: Если восемь человек строят стену за десять часов, сколько времени потребуется четырем мужчинам?

    Нет времени, потому что стена уже построена.

  9. Вопрос: Если у вас есть миска с шестью яблоками и вы берете четыре, сколько у вас есть?

    4, которые вы забрали

  10. Вопрос: Если бы у вас была только одна спичка и вы вошли в темную комнату с масляной лампой, дровами для растопки и газетой, что бы вы зажгли в первую очередь?

    Матч

  11. Вопрос: Если вы пишете «сидеть в ванне» s-o-a-k, а вы пишете «смешная история» j-o-k-e, как вы пишете «яичный белок»?

    a-l-b-u-m-e-n или e-g-g w-h-i-t-e

  12. Вопрос: Законно ли мужчине жениться на сестре своей вдовы?

    Нет, но так как он мертв, это будет сложно сделать.

  13. Вопрос: Если бунгало миссис Джон полностью оформлено в розовых тонах, а стены, ковер и мебель — всех оттенков розового, какого цвета будет лестница?

    Лестницы нет, потому что в бунгало нет второго этажа.

  14. Вопрос: Как мальчик ударил свой футбольный мяч на десять футов, а затем он вернулся к нему сам по себе?

    Он поднял это.

  15. Вопрос: Как мог человек выйти на улицу под проливным дождем без защиты и не промокнуть волосы на голове?

    Он был лысым.

  16. Вопрос: Если электропоезд движется на север со скоростью 100 миль в час, а ветер дует на запад со скоростью 10 миль в час, в какую сторону дует дым?

    В электричке нет дыма.

  17. Вопрос: Как могло случиться, что каждый человек в авиакатастрофе погиб, но два человека выжили?

    Двое оставшихся в живых поженились.

  18. Вопрос: Что ломается и никогда не падает, а что падает и никогда не ломается?

    Дневной перерыв и наступление ночи

  19. Вопрос: В некоторых месяцах 31 день, в других 30 дней, но в скольких месяцах 28 дней?

    Во всех месяцах 28 дней.

  20. Вопрос: «Адвокат — мой брат», — засвидетельствовал бухгалтер. Но адвокат засвидетельствовал, что у него нет брата. Кто врет?

    Ни того, ни другого, потому что бухгалтером была его сестра.

  21. Вопрос: На ферме дяди Билла случился ужасный шторм, и все овцы, кроме семи, были убиты. Сколько овец еще живы?

    Семерка

  22. Вопрос: Как вы называете женщину, которая все время знает, где находится ее муж?

    Вдова

  23. Вопрос: На чем вы сидите, спите и чистите зубы?

    Стул, кровать и зубная щетка

  24. Вопрос: Что поднимается и опускается, но всегда остается на одном месте?

    Лестница

  25. Вопрос: Что поднимается вверх, но никогда не опускается?

    Возраст

  26. Вопрос: Что произошло, когда было изобретено колесо?

    Произошла революция.

  27. Вопрос: Что имеет голову, хвост, но не имеет тела?

    Монета

  28. Вопрос: Кто больше, мистер Биггер, миссис Биггер или их ребенок?

    Младенец, так как он немного Больше.

  29. Вопрос: Если вы сидите с чашкой на столе лицом на юг, а вы находитесь на северной стороне стола, с какой стороны находится ручка чашки?

    Независимо от того, в какую сторону повернута чашка, ручка всегда находится снаружи.

Вопросы об обмане разума

Трудные вопросы, на которые нужно ответить, бросают вызов даже самому умному человеку.Вы должны не торопиться, внимательно прочитать вопросы и подумать, прежде чем пытаться ответить. Будьте осторожны, потому что ваше первое предположение может оказаться неверным при ответах на эти вопросы с обманом разума.

  1. Вопрос: Мужчина проживает на 100 этаже многоквартирного дома. В дождливые дни он поднимается на лифте. Однако в солнечные дни он проходит половину пути, а остаток пути идет по лестнице. Почему?

    Мужчина невысокого роста и может дотянуться до кнопки 50-го этажа лифта.В дождливые дни он пользуется ручкой зонтика.

  2. Вопрос: Мужчина и его сын попали в автомобильную аварию. Мужчина скончался по дороге в больницу, но мальчика срочно доставили в операцию. Хирург отделения неотложной помощи сказал: «Я не могу оперировать, потому что это мой сын!» Как это было возможно?

    Хирургом отделения неотложной помощи была его мать.

  3. Вопрос: Мужчина во всем черном идет по проселочной дороге. Внезапно из-за угла появляется большая черная машина без света и с визгом останавливается.Как водитель машины узнал, что он там?

    Было дневное время.

  4. Вопрос: Роберт, мой сосед, сорокапятилетний кузнец, ростом семь футов, ест весь день. Что он весит?

    Утюг

  5. Вопрос: Заполните эту серию чисел: 9 = 4, 21 = 9, 22 = 9, 24 = 10, 8 = 5, 7 = 5, 99 = 10, 100 = 7, 16 =?, 17 знак равно

    16 = 7 и 17 = 9 [(каждое число равно количеству букв в их написании, т. Е. 16 (шестнадцать) равно 7, а 17 (семнадцать) — 9.]

  6. Вопрос: В доме три важных комнаты .Первый заполнен деньгами. Второй заполнен важными бумагами. Третий наполнен украшениями. Однажды все эти комнаты загорелись. В каком помещении милиционеры потушили пожар в первую очередь?

    Ни одного, потому что полицейские не тушат пожары; пожарные делают.

  7. Вопрос: Почему 1968 пенни стоит больше, чем 1967?

    Потому что в пенни 1968 года на один пенни больше, чем в пенни 1967 года.

  8. Вопрос: У мужчины есть лиса, курица и мешок с зерном.Он должен перейти реку, но не может нести их всех сразу. Если он оставит лисицу с цыпленком, лиса съест цыпленка. Однако, если он оставит курицу вместе с зерном, курица съест зерно. Как он может безопасно перебраться через всех троих?

    Он может сначала захватить курицу. Затем он может вернуться и принести зерно, а затем нести курицу в эту поездку. Он оставляет курицу и заставляет лису ждать с зерном. Затем он возвращается на другую сторону и забирает курицу.

  9. Вопросы: Восемь мужчин сидят на диване. Три ноги ломаются, уходят шестеро мужчин. Сколько ног осталось?

    Five; ноги двух оставшихся мужчин (4) и оставшаяся нога кушетки.

  10. Вопрос: Вы ведете автобус. Когда вы начинаете свой маршрут, в автобусе едут пожилая женщина по имени миссис Смит и мальчик по имени Реймонд. На первой остановке старуха уходит, и входит продавец по имени Эд. На следующей остановке садятся Джек и его сестра Джилл, а также три женщины с сумками для покупок.Автобус идет пятнадцать минут, затем останавливается, выходит Реймонд, и в него садятся мужчина и его жена. Далее в автобус садится женщина с птицей в клетке. Как зовут водителя автобуса?

    Что ж, помните, что это вы водите автобус!

  11. Вопрос: Какое максимальное количество раз одну страницу газеты можно сложить пополам вручную?

    Только один раз, потому что после этого вы будете складывать его на четверти, восьмые и т. Д.

  12. Вопрос: Рыболовная лодка с лестницей прислонена к стене в гавани.В траулере 5 весел и 2 рыболовные сети. Расстояние между двумя последовательными ступенями лестницы — 1 метр. Если волны, ударяющиеся о стену, поднимаются на полметра каждые полчаса, сколько времени потребуется, чтобы 6 ступенек лестницы оказались под волнами?

    Поскольку рыбацкая лодка поднимается вместе с волнами, ни одна ступенька не уйдет под воду.

  13. Вопрос: Вы спрашиваете продавца в хозяйственном магазине: «Сколько будет стоить одна?» «Двадцать центов, — отвечает она. — И сколько мне обойдутся двенадцать?» ты спрашиваешь.«Сорок центов, — отвечает она. — Хорошо, я возьму сто двенадцать». Сколько вы заплатили?

    Ответ: Шестьдесят центов, потому что вам нужно было три числа для номера вашего дома — 112.

  14. Вопрос: Воскресным утром была убита старшая девочка в семье. Отец читал газету, мать готовила завтрак на кухне, а брат девочки играл в видеоигры. Дядя Джордж был в гостях и забирал почту. Кто убил девушку?

    Дядя Джордж, потому что это было воскресенье, а почта не доставляется в воскресенье.

  15. Вопрос: Если вы разделите 30 пополам и прибавите десять, что вы получите?

    70. Половина 0,5. Если вы разделите 30 на 0,5, вы получите 60. Добавьте десять, и вы получите 70.

Забавные вопросы-трюки

Некоторые вопросы-уловки вызывают смех, потому что они такие глупые. Детям особенно понравятся наши забавные вопросы-трюки. Мы включили несколько старых фаворитов, чтобы дети чувствовали себя умными. Задайте эти забавные вопросы своим друзьям или попробуйте ответить на них сами. Не ищите ответов, пока не дадите на каждый вопрос наиболее точный ответ.

  1. Вопрос: Что один костер сказал другому?

    Пойдем сегодня вечером?

  2. Вопрос: Почему нельзя похоронить женщину, живущую в Европе, в Канаде?

    Живую женщину нигде похоронить нельзя.

  3. Вопрос: Почему кинозвезды считаются крутыми?

    Потому что у них так много поклонников.

  4. Вопрос: Что такое белое и черное, а весь красный?

    Газету «читают» повсюду.

  5. Вопрос: В чем самая большая проблема с зимними ботинками?

    Снежные сапоги тают

  6. Вопрос: Почему гудят люминесцентные лампы?

    Потому что они не могут вспомнить слова.

  7. Вопрос: Чем больше высыхает, тем влажнее становится?

    Полотенце

  8. Вопрос: Что океан сказал пляжу?

    Ничего; он помахал.

  9. Вопрос: Где океан без воды?

    На карте

  10. Вопрос: Какой тип волос у океана?

    Волнистый

  11. Вопрос: Почему клоун пошел к врачу?

    Потому что он чувствовал себя немного смешно.

  12. Вопрос: Что такое липкое и коричневое?

    Палка

  13. Вопрос: Что будет, если белую шляпу бросить в Черное море?

    Шляпа намокает.

  14. Вопрос: Что дает силу и мощь проходить сквозь стены?

    Дверь

  15. Вопрос: Почему Супермен перешел дорогу?

    Он хотел попасть в супермаркет.

  16. Вопрос: Какой гвоздь старается пропустить плотник?

    Его ноготь

  17. Вопрос: Что вы всегда видите в начале парада?

    Письмо р

  18. Вопрос: Что едят снеговики на завтрак?

    Frosted Flakes

  19. Вопрос: Что случилось с человеческим пушечным ядром?

    Его уволили.

  20. Вопрос: В каком помещении нет стен?

    Гриб

  21. Вопрос: Как может женщина восемь дней обходиться без сна?

    Она спит по ночам.

  22. Вопрос: Как можно уронить сырое яйцо на бетонный пол и не расколоть его?

    Нет проблем, потому что бетонные полы очень трудно взломать.

  23. Вопрос: Как поднять слона одной рукой?

    Вы не можете, потому что вы никогда не найдете слона с одной рукой.

  24. Вопрос: Если у двух волн есть гонка, кто победит?

    Они «привязаны» (прилив).

  25. Вопрос: Почему ковбой приехал в город на своем муле?

    Потому что его мул был слишком тяжел, чтобы его нести.

  26. Вопрос: Что движется вверх и вниз, но не движется?

    Лестница

  27. Вопрос: Что случилось со львом, когда он съел клоуна?

    Он чувствовал себя забавным.

  28. Вопрос: Если петух кладет яйцо на крышу сарая, в какую сторону оно катится?

    Не катится, потому что петухи не откладывают яйца.

  29. Вопрос: Сколько животных взял Моисей в ковчег?

    Нет, потому что Ной построил и погрузил ковчег.

  30. Вопрос: Сколько ног у слона, если считать его хобот ногой?

    Четыре, потому что называть хобот слона ногой еще не значит.

  31. Вопрос: Сколько грязи в яме глубиной 3 фута, длиной 6 футов и шириной 4 фута?

    Нет, потому что внутри отверстия нет грязи.

  32. Вопрос: Что лоскутное одеяло сказало кровати?

    Я тебя прикрыл.

  33. Вопрос: Почему женщина ночью бегала вокруг своей кровати?

    Чтобы наверстать упущенное

  34. Вопрос: Что нельзя есть на завтрак?

    Обед или ужин

  35. Вопрос: При чем тут 4 колеса и мухи?

    Мусоровоз

  36. Вопрос: Сколько сторон у круга?

    Два, потому что у него есть внутренняя и внешняя стороны.

  37. Вопрос: Если вы представите, что находитесь в тонущей лодке в окружении акул, как бы вы выжили?

    Хватит воображать.

  38. Вопрос: Если десять птиц сидят на дереве и охотник стреляет в одну, сколько птиц останется на дереве?

    Нет, потому что после отстрела одной птицы остальные улетели.

  39. Вопрос: Если обезьяна, белка и птица мчатся на вершину кокосовой пальмы, кто первым получит банан?

    Нет, потому что вы не можете получить банан с кокосовой пальмы.

  40. Вопрос: Какие два слова в сочетании содержат больше всего букв?

    Почтовое отделение

Трюковые вопросы с ответами

Трюковые вопросы развлекают и информируют. Используйте нашу подборку для вечеринок, игровых вечеров — в любое время, когда вы собираетесь вместе с друзьями или коллегами. Некоторые из самых умных вопросов-уловок связаны с игрой слов, и в этот список мы включили немало. Вы найдете 30 вопросов, за которыми следует список ответов. На сколько вы можете правильно ответить?

Вопросы:

  1. Как дверь может не быть дверью?
  2. Какое слово начинается с IS , заканчивается на И и имеет в середине LA ?
  3. Почему на вечеринках всегда можно найти гриб?
  4. Хотите послушать анекдот про калий?
  5. Вы знаете какие-нибудь хорошие анекдоты про натрий?
  6. Что делать с мертвым химиком?
  7. Какое имя для юриста?
  8. Как вы называете человека в яме?
  9. Какая самая быстрая страна в мире?
  10. Что вы получите, если выбросите в океан миллион книг?
  11. Какая самая холодная страна в мире?
  12. Почему на дом пошел к врачу?
  13. На каких дорогах задерживаются призраки?
  14. Как вы называете женщину, стоящую на теннисном корте?
  15. Что круглого и чрезвычайно жестокого?
  16. Что случилось с крохотной гадалкой, сбежавшей из тюрьмы?
  17. Что зеленый и поет?
  18. Что случилось, когда фокусник рассердился?
  19. Какое распространенное слово из одиннадцати букв всегда пишется неправильно?
  20. Что выиграл изобретатель дверного молотка?
  21. Что случилось с женщиной, которая упала в обивочную машину?
  22. Вы слышали, что случилось с магазином оригами?
  23. Почему банкир бросил работу?
  24. Что вы найдете в глуши?
  25. Почему все так устали в первое апреля?
  26. Что один комод сказал другому комоду?
  27. Нейтрон заходит в кафе, заказывает завтрак и спрашивает официанта: «Сколько это будет?»
  28. Что встает, а растет вниз?
  29. Фотон заселился в гостиницу.Что она отвечает, когда ее спрашивают, нужна ли ей помощь с багажом?
  30. Если у отца Джорджа четверо детей — Ини, Мини и Миней, как зовут его четвертого ребенка?

Ответы:

  1. Когда это банка. (ajar = «open»)
  2. Island
  3. Потому что он гриб («веселый парень»).
  4. К! ( K — символ калия.)
  5. Na! ( Na, — символ натрия.)
  6. Барий («Похороните его»)
  7. Сью
  8. Дуг («выкопанный»)
  9. Россия («раш-я»)
  10. Волна титула
  11. Чили
  12. Потому что у него было оконное стекло («боль»).
  13. Тупики
  14. Аннет («сеть»)
  15. Порочный круг
  16. В целом он был мелким и средним.
  17. Элвис Петрушка
  18. Он вытащил своего зайца («волосы»).
  19. Неправильно
  20. Нобелевская премия
  21. Она полностью выздоровела.
  22. В сложенном виде.
  23. Он потерял интерес.
  24. Письмо H
  25. Потому что они только что завершили марш из 31 дня.
  26. Вы покраснели.
  27. Официант отвечает: «Тебе? Бесплатно!»
  28. Свеча
  29. Нет, спасибо, я путешествую налегке.
  30. Джордж

Вопросы-трюки стимулируют мозг и доставляют удовольствие. Они помогают заполнить время и расслабиться. Вдобавок они тренируют ваш мозг! Их любят все возрасты. Используйте хитрые вопросы и головоломки, чтобы сблизить сотрудников в офисе, избежать скуки и бросить вызов своим друзьям. Удачи, задавая наши каверзные вопросы!

Сьюзан изучала английский язык с двойным вторым по специальности «гуманитарные науки и бизнес» в Университете штата Аризона и получила степень магистра управления образованием в Университете Либерти.Она преподавала с четвертого по двенадцатый классы как в государственных, так и в частных школах. Предметы включали английский язык, американскую и всемирную историю и географию, математику, землю и физические науки, Библию, информационные технологии и творческое письмо.

Сьюзен писала-фрилансером более десяти лет, за это время она написала и отредактировала книги, газетные статьи, биографии, книжные обзоры, руководства, описания районов для риэлторов, презентации Power Point, резюме и множество других проектов.

Читать биографию полностью

Сбивать с толку и сбивать с толку лучшие умы (ответы тоже)

Мы собрали 101 лучший вопрос-трюк, чтобы заставить вас работать. Помните, что их усложняет то, что очевидный ответ обычно неверен. Есть скрытый уровень сложности. Вы вынуждены думать не только о своем первом решении.

Тем не менее, хорошая новость заключается в том, что, разгадав несколько таких загадок, вы начинаете разбираться в них. Просмотрите этот список до конца, и вы, вероятно, будете готовы практически к любой сложности.

Если вы ищете хотя бы пару хороших головоломок, наша группа экспертов * определила десять лучших из них чуть ниже. Но более серьезные игроки захотят продвинуться вниз по всей этой странице. Удачи — она ​​вам точно может понадобиться!

лучших хитрых вопросов с ответами

1.) 12 дроздов сидели на ветвях дерева. Мужчина выстрелил из своего ружья в одного черного дрозда. Сколько осталось на дереве?
Нет. Остальные улетели.

2.) Как звали президента в 1992 году?
Такой же, как сегодня.

3.) У матери Джона было трое детей. Первого ребенка назвали Эйприл. Второго ребенка назвали Мэй. Как звали третьего ребенка?
Джон

4.) Какая гора была самой высокой в ​​мире до открытия Эвереста?
Гора Эверест

5.) Представьте, что вы падаете в глубокую яму, полную ядовитых пауков и змей. Как бы вы выжили?
Хватит воображать.

Вопросы для трюка с разумом

6.) Если бы вы участвовали в гонке и обогнали человека, занявшего второе место, на каком месте вы были бы сейчас?
Второе место. (Если вы обгоните человека в первую очередь, вы окажетесь первым.)

7.) Мальчик пинает футбольный мяч. Он идет на 10 футов и возвращается к нему. Как это возможно?
Он подбросил его в воздух прямо над собой. Сила тяжести!

8.) Водитель грузовика идет по улице с односторонним движением не в том направлении и проезжает мимо не менее пяти полицейских.Почему его не поймали?
Он идет по тротуару, а не за рулем.

9.) Как можно поднять слона одной рукой?
Не беспокойтесь об этом, вы его не найдете.

10.) Если бы у вас была только одна спичка и вы вошли в темную комнату с масляной лампой, газетой и дровами, что бы вы зажгли в первую очередь?
Матч.

Загадки с ответами

11.) Как вы называете женщину, которая все время знает, где ее муж?
Вдова.

12.) У владельца ранчо было 500 коров, и он сделал один выстрел, и все они погибли. Как он это сделал?
Панорамное фото.

13.) Если бросить красный камень в зеленое море, как он вернется?
Мокрая. (Не зацикливайтесь на этом.)

14.) Как заставить исчезнуть номер один?
Добавьте букву «G» вперед, и ее больше нет.

15.) Сколько сторон у круга?
Два. Внутри и снаружи.

16.) Что становится все больше и больше, чем больше от этого отнимается?
Отверстие

17.) Почему это противозаконно, если человека, живущего в Северной Каролине, похоронят в Южной Каролине?
Нельзя хоронить живого человека.

сложных вопросов с трюками

18.) Какие два слова в комбинации содержат больше всего букв?
Почта и офис.

19.) В Новой Шотландии нельзя сфотографировать человека с деревянной ногой. Почему нет?
Вам понадобится какая-нибудь камера.

20.) У вас есть две монеты, которые в сумме составляют 30 центов, и одна из них не четверть.Какие это две монеты?
Четверть и пятак — один не четверть, а другой.

21.) Произошла страшная авиакатастрофа. Каждый человек на борту погиб, но два человека выжили. Как это возможно?
Выжившая пара была жената.

22.) Сколько книг размером 6 на 6 дюймов вы можете поместить в контейнер (2 x 2 фута), чтобы он больше не был пустым?
Один, после этого уже не пусто.

23.) Какая буква английского алфавита летает, поет и кусает?
«В»

24.) Джон упал с лестницы высотой 20 футов. Тем не менее, он не пострадал ни в малейшей степени. Почему?
Он был на перекладине вплотную к земле.

Трудные вопросы для ответа

25.) Что ломается без удержания?
Обещание.

26.) Какое дерево можно носить в руке?
Ладонь.

27.) В некоторых месяцах 31 день, а в других 30 дней. Сколько у 28 дней?
Все 12 умеют.

28.) Какое слово в словаре почти всегда пишется неправильно?
Неправильно.

29.) Что общего у острова и буквы «т»?
Они оба посреди воды.

30.) Она вышла замуж за многих мужчин, но никогда не была замужем. Кто она?
Министр.

31.) Что в конце радуги?
Буква «ш.»

32.) Как лучше всего бросить яйцо на бетонный пол, не треснув его?
Просто брось.Яйцо, скорее всего, треснет, а пол — нет.

33.) Женщина во всем черном идет по улице города. Внезапно из-за угла появляется большая черная машина без света и с визгом резко останавливается. Как машина узнала, что женщина там?
Было светло.

34.) Можете ли вы написать врага тремя буквами?
FOE.

35.) Я насекомое, и первая часть моего имени — это имя другого насекомого.Что я?
BEEtle.

36.) На что нужно ответить, но никогда не возникает вопроса?
Дверной звонок.

Логические вопросы

37.) Столица Кентукки произносится как Луисвилл или Лювилль?
Ни то, ни другое. Столица — Франкфурт.

38.) Создатель этого не хочет; покупатель не использует его, а пользователь не видит. Что это?
Гроб.

39.) У кого 4 пальца и большой палец, но не живое?
Перчатка.

40.) На какой стороне курицы больше перьев?
Снаружи.

41.) Чем всегда все кончается?
Буква «г».

42.) Как может мужчина, который бреется несколько раз в день, все еще иметь длинную бороду?
Он парикмахер.

43.) Какая шерсть лучше всего подходит для мокрых покрытий?
Слой краски.

44.) Сколько букв в алфавите?
11 букв в «алфавите».”

Вопросы-загадки

45.) Как ты 20 дней не спишь?
Сон по ночам.

46.) Если бы у Бренны было 4 кучи песка, а у Сары 5 куч песка, и они сложили бы их все вместе, сколько было бы куч?
Одна большая куча!

47.) Учитель задала двум девочкам, которые выглядели точно так же, как следующие вопросы: а.) Вы из одной семьи? б.) У вас одни родители? в) Вы родились в один день? Они ответили правдиво «да» на эти вопросы и все же сказали правду, когда указали, что они не близнецы.Как это может быть?
Они были тройняшками.

48.) Как долго длится ответ на этот вопрос?
Как долго.

Непонятные вопросы

49.) У отца Джона было трое сыновей: Щелкунчик, Треск и…. кто?
Джон.

50.) Какие следующие три буквы в этой последовательности: OTTFFSS?
ЛОР. Один, два, три… Это первая буква каждого числа, считая от единицы.

51.) Джон — мясник.Он ростом 5 футов 10 дюймов и носит обувь 11 размера. Что он весит?
Мясо.

52.) Что дырявое, но все еще держит жидкость?
Губка.

53.) Человек утверждал, что обладает особыми способностями, говоря, что он может предсказать счет любого бейсбольного матча до того, как он начнется. И он был прав. Как он это сделал?
Счет 0 — 0 перед началом игры.

54.) Женщина подвозит машину к гостинице, а затем сообщает владельцу, что она банкрот.Почему?
Она играет в монополию.

Вопросы-подсказки для детей

55.) Какой следующий номер в последовательности: 8 5 4 9 1 7 6 10 3 _?
Два. Номера расположены в алфавитном порядке.

56.) Братья Бен и Боб пошли играть в шашки в парке. Каждый сыграл по 5 игр. Бен сказал своей матери, что выиграл 4 игры. Боб сказал своей матери, что выиграл 3 игры. Оба говорили правду. Как это было?
Они не играли друг против друга.

57.) В озере есть участок кувшинок. Каждый день нашивка увеличивается в размерах вдвое. Если заплатке потребуется 50 дней, чтобы покрыть все озеро, сколько времени потребуется, чтобы заплатка покрыла половину озера?
49 дн. Потому что, когда он удвоится в этот день, он покроет все озеро.

Веселые

58.) Почему шестеро боялись семерых?
Потому что семь, восемь [съели], девять.

59.) Что круглого и чрезвычайно жестокого?
Замкнутый круг.

60.) Как узнать, что часы голодны?
Возвращается на четыре секунды назад.

61.) Какое слово самое длинное в английском языке?
Улыбается, потому что от старта до финиша миля. Найдите более простой ответ на самое длинное слово.

62.) Как поймать уникальное животное?
Уникальный up on ‘em.

63.) Как поймать ручное животное?
Прирученный путь.

64.) Почему бегемоты носят красные кроссовки?
Чтобы спрятаться в вишневых деревьях.
Вы никогда не видели бегемота на вишневом дереве?
Посмотрите, насколько хорошо это работает?

65.) Что выиграл изобретатель дверного молотка?
Приз без колокольчика.

66.) Какой правильный? «Пингвины летят» или «Пингвин летят»?
Ни то, ни другое, пингвин не летает.

67.) Кто больше, мистер Биггер, миссис.Больше или их ребенок?
Их ребенок немного побольше.

Узнайте больше Веселых загадок.

Веселые головоломки

68.) Как слона положить в холодильник?
Откройте дверь, поместите слона и закройте дверь.

69.) Как поставить жирафа в холодильник?
Откройте дверь, вытащите слона, введите жирафа и закройте дверь.

70.) Король Лев устраивает вечеринку и приглашает всех животных.Присутствуют все животные, кроме одного. Который из?
Жираф. Он все еще в холодильнике.

71.) Вы получаете приглашение на вечеринку Короля Льва. В пяти милях от дома Льва нужно пересечь реку, известную своими крокодилами. Как вы переходите?
Переплыть. Помните, что на вечеринке присутствуют все животные, кроме жирафа.

72.) Веревка, привязанная к шее быка, имеет длину 10 футов, а его кормушка находится на расстоянии 15 футов. Как бык смог успешно поесть, не порвав веревку и не перерезав ее?
Прошел.Веревка не была привязана ни к чему, кроме шеи быка.

73.) Четыре футболиста стояли под одним зонтом и не промокли. Как такое могло быть?
Дождя не было.

Trick Questions Изображение

Мем

Заблокирующие перегородки

74.) Как мог Том выйти на улицу под проливным дождем без шляпы или какой-либо защиты и при этом не промокнуть волосы на голове?
Покажите ответ: Том был лысым.

75.) Ковбой ехал на муле в город. Почему?
Он не мог нести его.

76.) Если электричка движется на север со скоростью 100 миль в час, а ветер дует на запад со скоростью 10 миль в час, как дует дым?
В электричке нет дыма.

77.) Петух отложил яйцо на крыше сарая. В какую сторону он катился?
Он не катился, потому что петухи не откладывают яйца. Куры делают.

78.) Что идет вверх и вниз, но остается на том же месте?
Лестница.

79.) Сколько раз можно вычесть 10 из 100?
Однажды. В следующий раз вы вычтете 10 из 90.

80.) Самолет разбивается на границе США и Канады. Где хоронят выживших?
Выживших хоронят не… только мертвых.

81.) Какой президент изображен на 100-долларовой банкноте?
Нет ни одного. Бенджамин Франклин никогда не был президентом.

82.) Как далеко можно пройти в лес?
На полпути.После этого вы выходите из леса.

83.) Если есть, то не поделюсь. Если я поделюсь им, у меня этого не будет. Что это?
Секрет.

84.) У отца Тома пять сыновей по именам: Десять, Двадцать, Тридцать, Сорок и… угадайте, как звали бы пятого?
Том будет пятым сыном.

85.) Две матери и две дочери вышли поесть, все съели по кусочку пиццы, но было съедено только три ломтика. Как такое возможно?
В группу обедающих входили бабушка, ее дочь и дочь ее дочери.

Головоломки со словами

86.) Как вы это интерпретируете: «Ban ana»?
Банановый сплит.

87.) Как насчет этого: OdOoOmO

Домино.

88.) Попробуйте следующее: «время земли».
Земля до времени.

89.) А этот: nasandwich, nasandwich

Бутерброды с тунцом.

90.) Трудный… что это значит? R.P.I.

Серьезная ошибка.(Следует читать «RIP»)

91.) Более простой: возрастная красотка

Возраст до красоты.

92.) Я люблю это: pol mom ice

Свекровь.

93.) И еще один: HoRobod

Робин Гуд.

94.) Сложнее, чем вы думаете…

I / 8

Я переоценен.

95.) Можете ли вы назвать слово, которое произносится иначе, если первая буква написана с большой буквы?

Травы или польский.

96.) Какой здесь ответ? полдень хороший

Добрый день.

97.) Как насчет этого? | г | е | а | г |

Читать между строк.

98.) А это? Картинка в кле

В рассоле.

99.) Попробуйте это:

0
к.т.н.
BA.
МБА

Три градуса ниже нуля.

Сверхпрочный

100.) Если у вас три, у вас есть три.Если у вас два, у вас есть два. Но если он у вас есть, у вас его нет. Что это?
Выбор

Самый известный вопрос с трюками

101.) Греки верили, что египетский сфинкс поглотит всех путешественников, которые не смогли правильно ответить на загадку, которую он поставил: «Что это за существо, которое ходит утром на четырех ногах, в полдень на двух, а вечером — на трех? ? » Это отличный пример загадки What Am I Riddles

.

Греческий герой Эдип придумал победный ответ: «Человек.Это привело к смерти Сфинкса.

Итак, мораль этой истории такова: если вы знаете ответы на сложные загадки, у вас много силы. Теперь вы хотите вернуться и еще раз просмотреть приведенные выше ответы?

Тим Муди и Майк О’Халлоран

Наша * экспертная группа: Тим, писатель и изобретатель игрушек, и Майк, писатель и редактор «Поэта поздравительных открыток», всегда любили хорошие загадки, головоломки и вопросы с подвохом. Тем не менее, даже при составлении этой коллекции они более чем несколько раз были в тупике.

и т.д.

Вы находитесь на странице с хитрыми вопросами.

Возможно, вам понравятся эти страницы, связанные с загадками:

Для взрослых

Головоломки

Легкие загадки

Хорошо

Сложные вопросы

Для детей

Математические загадки и головоломки

Хитрый

100 сверхсложных хитрых вопросов, которые оставят вас в тупике на несколько дней

Мэрилин Нивс / Гетти

Хорошо, smartypants, вы можете подумать, что знаете ответы на большинство вопросов, но задавали ли вам вопрос с подвохом? В отличие от пустяковых вопросов, они не требуют больших знаний или опыта.Вопросы с подвохом, в общем, хитрые. Вся цель состоит в том, чтобы попытаться обмануть вас, предложив неправильный ответ. И, по правде говоря, мы все время от времени могли использовать бесполезное упражнение. Но если вы случайно выберете их из парка, вы почувствуете себя самым умным человеком.

Это также отличное упражнение для вашего мозга. Итак, если вы хотите потренировать свой ум, вы попали в нужное место. Вопросы с подвохом не только забавны, но и заставляют задуматься.И это развлечение для всей семьи! Ваши дети будут смеяться над этими глупыми головорезами, особенно когда они услышат ответы. Самое лучшее в этих вопросах-уловках то, что когда вы думаете, что имеете правильный ответ, он оказывается даже проще, чем вы думали. Эти запросы являются титанами обмана и так что очень весело.

СВЯЗАННЫЙ: 100+ хитрых скороговорок для вашего милого маленького лингвиста

Заинтригованы? Мы собрали несколько лучших и забавных вопросов для вашей следующей семейной игры, которые наверняка заставят вас почесать голову.Как только вы пройдете через них (потому что мы знаем, что механизмы вашего мозга уже крутятся!), Вы можете попробовать наши сложные загадки, загадки для детей, загадки для подростков и «Что я?» загадки.

СВЯЗАННЫЙ: Тук-тук! Кто там? 60+ веселых анекдотов, которые сбивают вас с толку

  1. Маленькая девочка пинает футбольный мяч. Он идет на 10 футов и возвращается к ней. Как это возможно?
    Это называется гравитацией. Она подняла это.
  2. Какие две вещи нельзя есть на завтрак?
    Обед и ужин.
  3. Что всегда приходит, но никогда не приходит?
    Завтра.
  4. Что становится влажнее, чем больше сохнет?
    Полотенце.
  5. Как звали президента в 1992 году?
    Такой же, как сегодня.
  6. Каким словом вы бы описали человека, у которого не все пальцы на одной руке?
    Нормально, потому что люди обычно держат половину пальцев на одной руке.
  7. Какое слово написано неправильно в каждом словаре?
    Неправильно.
  8. Что общего у острова и буквы «т»?
    Они оба находятся посреди воды.
  9. Что можно сломать, но нельзя удерживать?
    Обещание.
  10. Что поднимается и опускается, но не может двигаться?
    Лестница.
  11. У меня есть зубы, но я не могу есть. Что я?
    Расческа.
  12. Что идет вверх, но никогда не опускается?
    Ваш возраст.
  13. Мужчина во всем черном идет по проселочной дороге. Внезапно из-за угла появляется большая черная машина без света и с визгом останавливается. Как машина узнала, что он там?
    Было дневное время.

    20th Century Fox

  14. Какое дерево можно носить в руке?
    Ладонь.
  15. Что живет, если его накормить, и что умирает, если его напоить?
    Огонь.
  16. У одного владельца ранчо было 500 коров, и он сделал один выстрел, и все они погибли. Как он это сделал?
    Панорамное фото.
  17. Петух отложил яйцо на крыше сарая. В какую сторону он катился?
    Не катился — петухи не откладывают яйца.
  18. Девушка упала с 50-футовой лестницы, но не пострадала. Как придешь?
    Она упала с нижней ступеньки.
  19. Что начинается с «е» и заканчивается «е», но содержит только одну букву?
    Конверт.
  20. Электропоезд движется на север со скоростью 100 миль в час, а ветер дует с запада со скоростью 10 миль в час. Куда дует дым?
    В электричке нет дыма.
  21. Как может человек восемь дней не спать?
    Во сне ночью.
  22. Как можно бросить сырое яйцо на бетонный пол, не треснув его?
    Яйцо не треснет бетонный пол!
  23. Что можно поймать, что не брошено?
    Холодно.
  24. Как заставить исчезнуть номер один?
    Добавьте букву «g» вперед, и ее больше нет.
  25. Что, если у вас есть, вы хотите поделиться со мной, а если вы поделитесь, у вас нет?
    Секрет.
  26. Если самолет упадет на границе США и Канады, где хоронят выживших?
    Выжившие не похоронены.
  27. Что больше Бога и зла больше дьявола. Богатые люди этого хотят, бедные люди этого хотят. А если съесть, умрешь?
    Ничего.
  28. В пятницу в город приехал ковбой. Он пробыл в городе три дня и уехал в пятницу.Как такое возможно?
    Пятница звали его лошадь.
  29. Какие два ключа не могут открыть дверь?
    Обезьяна и осел.
  30. Что на самом деле вы найдете в конце каждой радуги?
    Буква «ш».
  31. Как поднять слона одной рукой?
    Это не проблема, так как слона с одной рукой никогда не встретишь.
  32. Почему это противозаконно, если человека, живущего в Северной Каролине, похоронят в Южной Каролине?
    Нельзя хоронить живого человека.
  33. Как может девушка прожить 25 дней без сна?
    Она спит по ночам.
  34. Вы видите лодку, полную людей, но на борту нет ни одного человека. Как такое возможно?
    Все на борту женаты.
  35. Что тяжелее: 100 фунтов камней или 100 фунтов перьев?
    Они оба весят одинаково — 100 фунтов.
  36. Как мог человек выйти на улицу под проливным дождем без защиты и не промокнуть волосы на голове?
    Он был лысым.
  37. «Адвокат — мой брат», — засвидетельствовал бухгалтер.Но адвокат засвидетельствовал, что у него нет брата. Кто врет?
    Ни того, ни другого, потому что бухгалтером была его сестра.
  38. У чего есть голова и хвост, но нет тела?
    Монета.
  39. Произошла ужасная авиакатастрофа, все находившиеся на борту погибли, но два человека выжили. Как это возможно?
    Выжившая пара была жената.
  40. Если рыб 12 и половина из них тонет, сколько их?
    Их 12. Рыба не тонет!
  41. Если восемь человек строят стену по десять часов, сколько времени потребуется четырем мужчинам?
    Нет времени, потому что стена уже построена.
  42. Если у вас есть миска с шестью яблоками и вы берете четыре, сколько у вас есть?
    Четыре, которые вы забрали.
  43. Что принадлежит вам, но используется другими больше, чем вами?
    Ваше имя.
  44. Водитель грузовика едет по улице с односторонним движением не в том направлении и проезжает мимо как минимум десяти полицейских. Почему его не поймали?
    Потому что он не был за рулем! Он идет по тротуару.
  45. Представьте, что вы находитесь в комнате, которая быстро наполняется водой. Нет ни окон, ни дверей.Как ты выберешься?
    Хватит воображать.
  46. Это нужно всем в мире, но обычно они дают, не беря. Что это?
    Совет.
  47. Что можно держать, вообще не касаясь?
    Разговор.
  48. 10-футовая веревочная лестница свисает с борта лодки так, чтобы нижняя ступенька находилась на поверхности воды. Ступеньки находятся на расстоянии одного фута друг от друга, а скорость прилива составляет 6 дюймов в час. Сколько времени пройдет, пока не будут пройдены три ступеньки?
    Никогда.Лодка поднимается по мере приближения.
  49. Мальчик был доставлен в отделение неотложной помощи больницы, но врач скорой помощи осмотрел мальчика и отказался оперировать. «Этот мальчик — мой сын, — сказал доктор. Но доктор не был отцом мальчика. Как это могло произойти?
    Доктором была мама мальчика.
  50. У чего есть лицо и две руки, но нет рук или ног?
    Часы.

    Дисней

  51. Что правильно? «Пингвины летят» или «Пингвин летят».
    Нет, потому что пингвины не летают.
  52. В одних месяцах 31 день, в других 30 дней. Сколько у 28 дней?
    Во всех месяцах 28 дней.
  53. Что имеет голову, хвост, но не имеет тела?
    Монета.
  54. Я легкая как перышко, но даже самая сильная девушка не может удержать меня больше 5 минут. Что я?
    Дыхание.
  55. Мистер Джонс заметил, что карманы его брюк были совершенно пусты, но что-то в них все же было. Что это было?
    Отверстие.
  56. У кого четыре ноги, но не может ходить?
    Стол.
  57. Если бы у вас была только одна спичка и вы вошли в темную комнату с масляной лампой, дровами и газетой, что бы вы зажгли в первую очередь?
    Матч.
  58. Если восемь человек строят стену за десять часов, сколько времени потребуется четырем мужчинам, чтобы построить ее?
    Совсем нет времени, так как он уже построен.
  59. Что становится резче, чем больше вы его используете?
    Твой мозг.
  60. Что можно бегать, но не ходить?
    Капли дождя.
  61. Если павлин мистера Смита откладывает яйцо в мистера Смита.Двор Джонса, кому принадлежит яйцо?
    Павлины не откладывают яиц, только павлины.
  62. A является отцом B. Но B не является сыном A. Как это возможно?
    Б — дочь.
  63. Я полон дыр, но могу сдерживать воду. Кто я?
    Губка.
  64. Почему 1968 пенни стоят больше, чем 1967?
    В пенни 1968 года на один пенни больше, чем в пенни 1967 года.
  65. Какое максимальное количество раз одну страницу газеты можно сложить пополам вручную?
    Только один раз, потому что после этого вы будете складывать его на четверти, восьмые и т. Д.
  66. Что один костер сказал другому?
    Пойдем сегодня вечером?
  67. Как узнать, что часы голодны?
    Возвращается на четыре секунды назад.
  68. Как долго длится ответ на этот вопрос?
    Как долго.
  69. Какая шерсть лучше всего ложится во влажном состоянии?
    Слой краски.
  70. Как может человек, бреющийся несколько раз в день, иметь длинную бороду?
    Он парикмахер.
  71. Что хочет получить ответ, несмотря на то, что никогда не задавал вопросов?
    Дверной звонок.
  72. Можете ли вы написать врага, используя только три буквы?
    Ф-О-Е.
  73. Сколько сторон у круга?
    Два, внутри и снаружи.
  74. Если вице-президент умирает, кто действует как президент?
    Президент.
  75. Какая гора была самой высокой на земле до открытия Эвереста?
    Гора Эверест.
  76. Сколько раз можно вычесть 10 из 100?
    Один раз, потому что в следующий раз из 90 будет вычтено 10.
  77. Что имеет много ключей, но не может открыть дверь?
    Фортепиано.
  78. Если бы вы бросили красный камень в зеленое море, как бы он вернулся?
    Мокрая.
  79. В каком помещении нет стен?
    Гриб.
  80. Почему женщина бегала ночью вокруг своей кровати?
    Чтобы наверстать упущенное.
  81. Две матери и две дочери вышли поесть, все съели по кусочку пиццы, но было съедено только три ломтика. Как такое возможно?
    В состав группы входили бабушка, ее дочь и дочь ее дочери.
  82. Какое пятибуквенное слово становится короче, если к нему добавить две буквы?
    Короткий.
  83. Можете ли вы назвать три дня подряд, не используя слов среда, пятница и воскресенье?
    Вчера, сегодня и завтра.
  84. Что происходит раз в минуту, два раза в мгновение и никогда в тысячу лет?
    Буква М.
  85. Во-первых, вы выбросите мою внешность и приготовьте внутри. Затем вы съедите мою внешность и выбросите мою внутреннюю часть. Что я?
    Кукуруза в початках, потому что вы выбрасываете шелуху и готовите кукурузу. Затем вы едите ядра и выбрасываете початки.
  86. Если вы пишете «сидеть в ванне» как «s-o-a-k», а «забавная история» как j-o-k-e, как вы пишете «яичный белок»?
    E-g-g-w-h-i-t-e.
  87. Что произошло, когда было изобретено колесо?
    Произошла революция.
  88. Что вы скажете, когда наконец что-то поймете?
    «Ой».
  89. Сколько кирпичей нужно, чтобы построить кирпичное здание?
    Один кирпич.
  90. У отца Марии пятеро дочерей по имени Тана, Тене, Тини, Тоно.Как зовут пятую дочь?
    Мэри.
  91. Я всегда в движении, во мне живут рыбы, во мне можно плавать, но я меньше других. Что я?
    Река.
  92. Вы играете меня, дергая за струны, но никогда не касаетесь моих струн. Что я?
    Фортепиано.
  93. Если бы у Салли было четыре кучи песка, а у Джуди было бы пять куч песка, и они сложили бы их все вместе, сколько было бы куч?
    Одна действительно большая куча.
  94. Сколько букв в алфавите?
    Одиннадцать букв в «алфавите».”
  95. Запрещается фотографировать человека с деревянной ногой. Почему?
    Потому что деревянные ножки не умеют фотографировать! Вам понадобится камера.
  96. Кто похоронен в могиле Гранта?
    Никто. Грант погребен, а не похоронен.
  97. Учитель задает двум абсолютно одинаковым девочкам следующие вопросы: Вы из одной семьи? У тебя такие же родители? Вы родились в один день? Девочки ответили утвердительно на все вопросы, но, в конце концов, они не были близнецами.Как это возможно?
    У них была еще одна сестра, и вместе у них родились тройня.
  98. Я — слово, начинающееся с буквы «i». Если вы добавите ко мне букву «а», я стану новым словом с другим значением, но оно будет звучать точно так же. Какое я слово?
    Остров.
  99. Вы видите меня один раз в июне, два раза в ноябре и совсем не в мае. Что я?
    Буква «е».
  100. Если двое — компания, а трое — толпа, то какие четыре и пять?
    Девять.

151 хитрый вопрос для детей (каверзные вопросы с ответами) Детские занятия

Раскрытие информации: Этот пост может содержать партнерские ссылки, что означает, что я получаю комиссию, если вы решите совершить покупку по моим ссылкам бесплатно для вас.Как партнер Amazon, я зарабатываю на соответствующих покупках. Прочтите полное раскрытие здесь.

Вопросы-подсказки для детей

Мы собрали для детей 151 разный вопрос-трюк, над которыми интересно поразмышлять. Чтобы помочь детям восстановить связь со своей способностью сосредоточиться и глубже погрузиться в богатство творчества, просто задайте один из этих сложных вопросов и дайте им время подумать.

Не беспокойтесь о неправильных ответах на вопросы. Было очень весело услышать неправильные ответы на некоторые вопросы.Часто вопросы с подвохом формулируются таким образом, чтобы получить неправильные ответы. Убедитесь, что вы создали безопасное и веселое место, где можно ошибаться.

Попробуйте эти маленькие уловки разума, когда у вас появится шанс с детьми в вашей жизни, но не удивляйтесь, если вы обнаружите, что немного почесываете голову с некоторыми загадками, головоломками и вопросами-трюками, которые мы выделили ниже. , или!

хитрых вопросов с ответами

1. Мистер Джонс заметил, что карманы его брюк были совершенно пусты, но что-то в них все же было.Что это было?
Ответ

Дырка.

2. Какие буквы легко найти в каждый день недели?
Ответ

Д — А — Я

3. Какое животное из джунглей прячется «внутри» этих букв — L P H N?
Ответ

Слон.

4. Что останется, если вынуть муравья из подорожника?
Ответ

Нечто незамысловатое.

5. Мышь хочет поправиться.Она собирается начать с подъема по лестнице. Начиная с четвертого этажа, она поднимается на пять этажей, вниз на семь, на шесть этажей, на три этажа и снова на несколько этажей. На каком она этаже?
Ответ

Девятый этаж.

6. Можете ли вы добавить один математический символ из числа 55555, чтобы получилось 500?
Ответ

555-55 = 500

7. Адам повернулся налево, направо, развернулся на 360 °, встал на голову, а затем пошел назад, прежде чем сфотографировать закат.В каком направлении он указывал?
Ответ

Запад.

8. Семья Джонсов поехала на юг на летние каникулы, пройдя по пути через пять разных штатов. Брэдфорды едут в противоположном направлении и прошли всего три штата. В каком направлении они едут?
Ответ

Север

9. Белый медведь Петр устроил вечеринку по случаю дня рождения, каждый гость принес по пять рыбок. Гадкий маленький пингвин украл две рыбы, но осталось 198 штук.Сколько человек пришло на вечеринку?
Ответ

40.

10. Кролик Рики съедает 1 морковь в воскресенье, 2 моркови в понедельник, 4 моркови во вторник и так далее. Сколько моркови он ест каждую неделю?
Ответ

127 морковок.

11. Если пять аллигаторов могут съесть пять рыб за три минуты, сколько времени понадобится 18 аллигаторам, чтобы съесть 18 рыб?
Ответ

Три минуты.

12. Если завтра я сказал «позавчера была суббота», какой сегодня день?
Ответ

Воскресенье.

13. Лягушонок Фредди попал в аварию и сломал себе ногу. Он пошел в больницу в понедельник, 1 ноября, и уехал 30 ноября. Какой сейчас день недели?
Ответ

Вторник.

14. Мистер и миссис Банджо едят мороженое. Миссис Банджо говорит, что если муж даст ей одну шарик своего мороженого, у нее будет в два раза больше шариков, чем у него. Мистер Банджо говорит, что если она даст ему одну из имеющихся у нее совков, то получится столько же. Сколько ложек у каждого из них?
Ответ

ГоспожаУ Банджо семь совков, а у мистера Банджо пять.

15. Рози заказывает шесть тарелок мороженого по три шарика в каждой. Тони хочет столько же ложек, сколько и она, но всего в двух блюдах. Сколько ложек в каждой его посуде?
Ответ

Девять мерных ложек в каждой.

16. Лаура говорит, что она может бросить мяч так, что он улетит от нее, внезапно изменит направление, а затем вернется к ней сама. Это вообще возможно?
Ответ

Да, при условии, что она подбрасывает мяч прямо в воздух.

17. В мешочке мармеладных мишек три красных, три зеленых, три желтых и три белых. Сколько мармеладных мишек нужно вынуть из пакета, чтобы получить трех одинаковых по цвету?
Ответ

Девять.

18. В банке для печенья находится 10 печений с шоколадной крошкой и 10 печений M; M. Сколько файлов cookie нужно вынуть, чтобы получить по два каждого вида?
Ответ

12 куки.

19. Лиза — племянница Уильяма, но не племянница Эмили, хотя Эмили — сестра Уильяма, а Уильям не женат.Как такое возможно?
Ответ

Лиза — дочь Эмили.

20. Вы можете назвать три вещи, которые продаются только парами?
Ответ

Перчатки, обувь, носки.

21. У трех часов разное время. Один немного быстрее, один немного медленнее и один просто) одни часы показывают 10:05, другие 9:56 и еще 10:11. Что правильно?
Ответ

10:05

22. Было наименьшее количество монет, которое вы можете использовать, чтобы получить ровно 0 долларов.65
Ответ

Две четвертинки, десять центов и пятак.

23. Мэгги живет на улице с 10 домами. Дома пронумерованы от 1 до 10. Если Мэгги сложит все номера домов, меньшие, чем у нее, общая сумма будет в три раза больше ее фактического номера дома. Какой номер у дома Мэгги?
Ответ

Дом номер семь.

24. Выходит новый фильм, в котором первые люди появляются за час до начала показа. Публика удваивается каждые 10 минут, а через 60 минут театр заполняется.Когда театр был заполнен наполовину?
Ответ

За 10 минут до начала фильма.

25. Какие числа, кратные четырем, находятся между 10 и 19?
Ответ

12 и 16

26. Какие нечетные числа от 10 до 16?
Ответ

11, 13 и 15

27. Какое наибольшее число можно составить из чисел три, пять и семь?
Ответ

753

28.Каждый год на Марсе в два раза больше, чем на Земле. Сколько бы вам было лет, если бы вы жили на Марсе?
Ответ

Удвойте свой возраст.

29. Семь братьев родились с разницей в два года. Младшему брату семь лет. Сколько лет старшему брату?
Ответ

19.

30. Мистер Адамс ровно в 5 раз старше его пса Спайка. Через пять лет мистеру Адамсу будет всего в три раза старше Спайка. Сколько ему будет лет через пять?
Ответ

30.

31. Если бы в каждом месяце было ровно 30 дней, вместо некоторых 29, некоторых 30 и некоторых 31, каждый год был бы длиннее или короче?
Ответ

Короче (360 дней)

32. Какие два числа получатся одинаковыми при умножении или сложении их?
Ответ

2 × 2 = 4 и 2 + 2 = 4.

33. Рики и Билли находят салат с 20 отдельными листиками. Поскольку Рики первым нашел салат, он получил на один лист больше, чем его друг.Сколько листьев у Рики?
Ответ

10 ½ листов.

34. Если бы вы пересчитали все уши в стране и затем разделили это число на два, что бы вы узнали наверняка?
Ответ

Сколько живых существ было в той стране.

35. Учитель говорит, что в классе меньше 30 учеников, но больше 20 человек. Детей можно разделить на группы по два, три, четыре, шесть и восемь человек без остатка.Сколько там студентов?
Ответ

24 ученика.

36. Карла уезжает из Нью-Йорка в восемь утра, двигаясь со скоростью 35 миль в час. Харриет едет в Нью-Йорк со скоростью 50 миль в час. Когда они встретят, какой из них будет дальше всего от Нью-Йорка?
Ответ

Когда они встретятся, они будут на таком же расстоянии от Нью-Йорка.

37. Если дни на Марсе равны двум неделям на Земле, сколько времени длится час?
Ответ

14 земных часов.

38. Эми 14 лет, а ее матери 38. Сколько лет назад ее мать была ровно в 3 раза старше Эми?
Ответ

Два года назад.

39. Пекарь несет мешок с мукой весом 15 фунтов. Он достает половину муки, но обнаруживает, что мешок весит 9 фунтов. Насколько тяжелый пустой мешок?
Ответ

3 фунта.

40. Что говорится в этом секретном сообщении? Tha twasn treall ys oh ardaf terall
Ответ

В конце концов, это было не так уж и сложно.

41. Миссис Адамс говорит соседке, что ее сыну сегодня исполняется 16 лет, а в следующий день рождения ему исполнится 20. Как такое может быть?
Ответ

Родился 29 февраля високосного года.

42. Какой объект не входит в этот список — труба, флейта, скрипка, губная гармошка, туба?
Ответ

Скрипка.

43. Если вы бросили один кубик только один раз, с меньшей вероятностью вы выбросите нечетное число, число больше трех или число меньше трех?
Ответ

Число меньше трех.

44. В турнире по баскетболу участвуют 16 игроков. Сколько игр должно пройти, прежде чем будет объявлен победитель?
Ответ

15.

45. Фермер держит и гусей, и коз. Он посчитал ноги у всех своих животных и обнаружил, что их ровно 36. Сколько гусей и сколько коз у него?
Ответ

Гуси козам — 2/8, 4/7, 6/6, 8/5, 10/4, 12/3, 14/2

46. Если бы были записаны только эти буквы — J FM AM J JAS OND — что бы они значили?
Ответ

Первое письмо каждого месяца в году.

47. Четверо друзей участвуют в гонках. Лиза финиширует на четыре часа раньше Тони, а Пит — на восемь часов впереди Макса. Макс нужно было на шесть часов больше, чем Лизе. В каком порядке они пересекают финишную черту?
Ответ

Пит, Лиза, Тони, Макс.

48. День матери — это всегда второе воскресенье мая. Когда это может быть самый ранний день?
Ответ

8 мая.

49. Если вы родились в нечетный год, отметите ли вы свое 50-летие в нечетном или четном году?
Ответ

Нечетный год.

50. Билли говорит своей сестре Саре, что сейчас он вдвое старше и в два раза умнее ее. Сара говорит, что через пять лет она должна быть вдвое старше, чем сейчас, и что Билли уже не будет. Сколько им будет лет через пять?
Ответ

Билли будет 15, а Саре будет 10.

51. Вместе, Том и его отец ушли 280 фунтов. Папа Тома весит в три раза больше, чем он сам. Сколько весит Том?
Ответ

70 фунтов.

52. Какие два числа образуют однозначное число, когда вы их умножаете, и двузначное число, когда вы складываете их вместе?
Ответ

Один и девять.

53. Городской автобус отправляется утром без пассажиров. На первую остановку садятся два человека, на следующей — еще пять. Один за другим выходят семь человек и выходят три человека. На следующей остановке пять человек входят и шесть человек выходят. На последней остановке три человека входят и один человек выходит.Сколько остановок было сделано?
Ответ

Пять остановок.

54. Сколько существует двузначных чисел?
Ответ

92.

55. Как далеко олень может забежать в лес?
Ответ

На полпути!

56. Я нечетное число, но если убрать хотя бы одну букву, я стану четным. Вы можете угадать мой номер?
Ответ

Семь.

57. Что имеет руки, но не указывает, не отмахивается и никогда не хлопает в ладоши?
Ответ

Часы.

58. Если у вас есть красный дом из красных кирпичей и желтый дом из желтых кирпичей, из чего сделать теплицу?
Ответ

Стекло!

59. Что нельзя использовать, пока не сломаешь?
Ответ

Яйца.

60. Что такое то, что поднимается и поднимается, но никогда не упадет?
Ответ

Ваш возраст.

61. Можете ли вы угадать слово из пяти букв, которое становится короче, если к нему добавить две буквы?
Ответ

Короткий.(Короткое плюс ER равно короче)

62. Вы можете придумать слово, которое начинается с Т, заканчивается на Т и содержит Т?
Ответ

Чайник.

63. Я самый высокий в новеньком, но намного, намного ниже, когда изношен. Что я?
Ответ

Свеча.

64. Знаете ли вы, что одно слово написано неправильно в каждом когда-либо составленном словаре?
Ответ

Слово «неправильно».

65.Проведите линию на листе бумаги. Как можно сделать ее длиннее, не касаясь этой линии?
Ответ

Нарисуйте короткую линию рядом с ним.

66. Можете ли вы придумать что-нибудь настолько деликатное, что простое произнесение названия сразу его сломает?
Ответ

«Тишина».

67. Какое слово начинается с E, но состоит только из одной буквы?
Ответ

Конверт.

68. Можете ли вы придумать что-нибудь, во что действительно легко попасть, но от чего действительно трудно выбраться?
Ответ

Беда!

69.Вся семья Браунов не стояла бы под зонтиком, но все равно не промокла. Как такое возможно?
Ответ

Дождя сначала не было.

70. У мамы и папы Дэвида было трое сыновей. Их звали Snap, Crackle и… что?
Ответ

Дэвид.

71. У женщины по имени Маргарет четыре разных дочери, и у каждой из этих дочерей есть брат. Сколько всего детей у Маргарет?
Ответ

Пять.У каждой дочери есть один и тот же брат.

72. Произошла ужасная авиакатастрофа, и все люди погибли. Кто выжил?
Ответ

Все, кто был женат.

73. Можете ли вы придумать что-то с четырьмя глазами, но полностью слепой?
Ответ

Слово «Миссисипи».

74. Можете ли вы придумать что-то, что путешествует по всей планете, но всегда остается в одном и том же месте?
Ответ

Почтовая марка.

75. Сколько букв в английском алфавите?
Ответ

15 — В английском семь слов, а в алфавите восемь.

76. Есть слово из шести букв, которое оставляет 12 после того, как вы убираете одну из них. Что это за слово?
Ответ

Десятки.

77. Можете ли вы придумать что-то, что полностью дырявое, но все же удерживает много воды?
Ответ

Губка.

78.Есть вопрос, на который ты никогда не сможешь ответить утвердительно. Знаете ли вы, что это такое?
Ответ

Ты еще спишь?

79. Если все пойдет не так, совсем боком, на что вы всегда можете рассчитывать?
Ответить

Пальцы рук и ног!

80. Можете ли вы представить себе что-то, что всегда находится прямо перед вами, но никогда не может быть увидено или удержано?
Ответ

Будущее.

81. Вы знаете, где можно найти города и страны, поселки и магазины, дороги и озера, но совершенно никаких людей?
Ответ

На карте.

82. Вы знаете, почему призраки и духи делают таких ужасных лжецов?
Ответ

Потому что их легко увидеть насквозь!

83. Представьте себе это на секунду. Вы находитесь в комнате, и стены смыкаются, пол раскрывается под вами, а вода льется с потолка. Нет ни окон, ни дверей, чтобы сбежать. Как ты выберешься?
Ответ

Хватит воображать.

84. Можете назвать две вещи, которые НИКОГДА нельзя есть на завтрак?
Ответ

Обед или ужин.

85. Если вы войдете через синий или зеленый камень в Красное море, кем они станут?
Ответ

Намокание.

86. Что люди едят, но не едят на самом деле?
Ответ

Серебро и тарелки.

87. Вы можете придумать что-нибудь, что можно подавать, но нельзя есть?
Ответ

Волейбольный мяч.

88. Можете ли вы придумать что-нибудь, что вы всегда можете поймать, но никогда не сможете бросить?
Ответ

Грипп.

89. Знаете ли вы, что каждый день поднимается и опускается каждую минуту, но на самом деле никогда не движется?
Ответ

Температура.

90. Можете ли вы придумать что-нибудь, что начинается с P и заканчивается на E, но содержит тысячи букв?
Ответ

Почтовое отделение.

91. Если мистер Оранжевый живет в синем доме, мистер Желтый живет в зеленом доме, а мистер Красный живет в фиолетовом доме, который живет в Белом доме?
Ответ

Президент.

92. В одноэтажном доме синего цвета жили розовый человек, зеленая собака, желтая рыбка, фиолетовый компьютер, серебряный стул, золотой стол и невидимый телефон. Какого цвета была лестница?
Ответ

Нет.

93. Вы можете придумать что-нибудь, что становится все влажнее и влажнее, чем больше сохнет?
Ответ

Полотенце.

94. Можете ли вы придумать что-нибудь, что идет по всему лесу, но на самом деле никогда не выходит в лес?
Ответ

Кора дерева.

95. Если вы видите электричку, идущую на восток со скоростью 70 миль в час, но дует ветер с запада со скоростью 80 миль в час, в какую сторону будет дуть дым от поезда?
Ответ

Электропоезда не дымят.

96. У меня полно ключей, но нет замков. У меня есть место, но нет дополнительных комнат. Конечно, вы можете войти, но выхода нет. Что я?
Ответ

Клавиатура.

97.Можете ли вы представить себе что-то легкое, как перышко, но даже самый сильный человек в мире не смог бы удержать его дольше минуты?
Ответ

Твоё дыхание.

98. Можете ли вы придумать что-нибудь, что бежит, но никогда не ходит, имеет рот, но никогда не разговаривает, имеет голову, но никогда не плачет, есть кровать, но никогда не уснет?
Ответ

Река.

99. Знаете ли вы, как называется изобретение, которое позволяет мгновенно смотреть сквозь любую стену, какую только захотите?
Ответ

Окно.

100. У матери Меган пять разных дочерей. Одного зовут Пегги, одного зовут Донна, одного зовут Бетти, другого зовут Джоан. Как зовут пятую дочь?
Ответ

Меган.

101. Если самолет потерпит крушение на границе Англии и Шотландии, где хоронят выживших?
Ответ

Выживших не хороните.

102. Двенадцать груш висели высоко, двенадцать человек проходили мимо, каждый взял грушу и оставил одиннадцать висеть там.Как это может быть? Как же остаться одиннадцать груш?
Ответ

«Каждый» — это мужское имя!

103. Если есть лягушка, мертвая в центре кувшинок, прямо посреди пруда, на какую сторону она прыгнет?
Ответ

Ни то, ни другое, лягушка мертва!

104. Вы водитель автобуса. На первую остановку садятся 4 человека. На второй остановке садятся 8 человек, на третьей выходят 2 человека, на четвертой выходят все.Вопрос в том, какого цвета глаза водителя автобуса?
Ответ

Так же, как и ваш, вы водитель автобуса.

105. Что никогда не становится более влажным, независимо от того, сколько идет дождь?
Ответ

Море

106. Мужчина вышел под проливным дождем без защиты, но ни один волос на его голове не промок… как же так?
Ответ

Лысый был.

107. У отца Дэвида трое сыновей: Снап, Кракл и _____?
Ответ

Дэвид

108.У чего есть рот, но он не ест? Банк без денег? Кровать, но не спит — машет, а рук нет?
Ответ

Река

109. Ковбой приехал в гостиницу в пятницу. Он останавливался на две ночи и уехал в пятницу. Как такое могло быть?
Ответ

Его лошадь звали Пятница

110. Если красный дом находится с правой стороны, а синий — с левой стороны, то где белый дом?
Ответ

Вашингтон, округ Колумбия

111.Какие два слова в сочетании содержат больше всего букв?
Ответ

Почтовое отделение

112. Какое слово, написанное заглавными буквами, одинаково в обратном, обратном и перевернутом направлениях?
Ответ

ПОЛДЕНЬ

113. Какая точная середина Америки?
Ответ

Буква «р»

114. Что становится влажнее и влажнее, чем больше сохнет?
Ответ

Полотенце

115.Как далеко собака может забежать в лес?
Ответ

На полпути — когда он доходит до места, он уже на пути «наружу».

116. В бейсболе какие могут быть ауты в иннинге?
Ответ

По шесть человек в каждой команде по три

117. В лодке сидели десять подражателей. Выскочил один. Сколько осталось?
Ответ

Ни одного — они были «подражателями», так что все они выпрыгнули!

118. Какое самое длинное слово в английском языке?
Ответ

СМИЛИ — Потому что от начала до конца миля

119.Если есть три яблока и вы взяли два, сколько у вас есть?
Ответ

Два — потому что ВЫ взяли два.

120. Было красное бунгало. В нем все было красным — стены, ковры, мебель. Какого цвета были лестницы?
Ответ

Лестниц не было. Это было бунгало!

121. Какое второе имя Медведя Руперт?
Ответ

The!

122. У чего есть голова и хвост, но нет тела?
Ответ

Монета!

123.Если красный дом находится с правой стороны, а синий — с левой стороны, где белый дом?
Ответ

В Вашингтоне, округ Колумбия

124. Как сделать буквы в NEW DOOR, составить одно слово?
Ответ

ОДНО СЛОВО!

125. Что произошло в 1961 году и не повторится до 6009 года?
Ответ

Год снова будет тот же в перевернутом виде

126. Какая гора была самой высокой в ​​мире до открытия Эвереста?
Ответ

Mt.Эверест все еще был самой высокой горой — даже ДО того, как его обнаружили

127. Что было раньше, курица или яйцо?
Ответ

Динозавры откладывали яйца задолго до появления кур!

128. Как у 22-го и 24-го президентов США могли быть одни и те же родители, но они не были братьями?
Ответ

Гровер Севеланд дважды избирался на 22-й и 24-й президентские должности.

129. Назовите последний год, когда Новый год предшествовал Рождеству.
Ответ

В этом году Новый год всегда предшествует Рождеству того же года.

130. Если бы у вас была только одна спичка и вы вошли в темную комнату с масляной лампой, газетой и дровами, что бы вы зажгли в первую очередь?
Ответ

Матч

131. Если павлин мистера Смита откладывает яйцо во дворе мистера Джонса, кому принадлежит яйцо?
Ответ

Никто. Павлины не откладывают яиц; пави делают.

132.Почему пенни 1968 года стоят больше, чем пенни 1967 года?
Ответ

Так как пенни 1998 года на единицу больше, чем пенни 1997 года

133. Начните с трех монет: пенни, пятак и четверть. Скажите: «У матери Томми трое детей. Первого ребенка зовут Пенни. (Сказав это, поднимите пенни.) «Второго ребенка зовут Ник». (Поднимите пятак.) Как зовут третьего ребенка? (Поднимите четверть.)
Ответ

Томми

134.Как долго длилась Столетняя война?
Ответ

116 лет

135. В какой стране производятся панамские шляпы?
Ответ

Эквадор

136. От какого животного мы получаем кетгут?
Ответ

Овцы и лошади

137. Какого цвета черный ящик на коммерческом самолете?
Ответ

Оранжевый конечно

138. В каком месяце россияне празднуют Октябрьскую революцию?
Ответ

Ноябрь

139.Из чего сделана расческа для верблюжьей шерсти?
Ответ

Мех белки

140. В честь какого животного названы Канарские острова в Тихом океане?
Ответ

Собаки

141. Как звали короля Георга VI?
Ответ

Альберт

142. Какого цвета зяблик пурпурный?
Ответ

Малиновый

143. Откуда китайский крыжовник?
Ответ

Новая Зеландия

144.У меня полно ключей, но я не могу открыть ни одну дверь. Что я?
Ответ

Фортепиано

145. Я дырявый, но все еще могу удерживать много воды. Что я?
Ответ

Губка

146. Чем больше ты у меня забираешь, тем больше я становлюсь. Что я?
Ответ

Отверстие

147. У меня есть руки, но я не могу хлопать. Что я?
Ответ

Часы

148.Вы можете поймать меня, но не можете бросить. Что я?
Ответ

Холодное

149. Я спускаюсь, но никогда не поднимаюсь. Что я?
Ответ

Дождь

150. Я мяч, который можно катать, но нельзя отбрасывать или бросать. Что я?
Ответ

Глаз

151. Наполни меня горячим или холодным. Вложи что-нибудь в меня, и я обязательно буду держать. Что я?
Ответ

Чашка

Заключительные мысли

Мы надеемся, что вам понравились эти вопросы-трюки, загадки «Что я» и головоломки для детей так же весело, как и нам.

Нет лучшего способа развить умы молодых людей, чем задавать им веселые головоломки, викторины и вопросы с подвохом. Помогите им подумать немного усерднее и глубже и глубже погрузиться в свое творчество.

Попробуйте пару из них на молодых в вашей жизни и посмотрите, насколько весело они пытаются понять их!

Список литературы

https://www.prodigygame.com/blog/brain-teasers-for-kids/
https: // brightside.мне / чудо-викторины / 17-сложных-головоломок-для-детей-которые-растягивают-ваш-мозг-1

/
https://www.everythingmom.com/parenting/45-riddles-and-brain-teasers -для детей
https://icebreakerideas.com/brain-teasers/
https://frugalfun4boys.com/30-riddles-brain-teasers-for-kids/
https://www.fatherly.com/play / the-best-riddles-for-kids-not-confusing /
https://www.everythingmom.com/parenting/45-riddles-and-brain-teasers-for-kids
https://www.allprodad. com / 10-great-riddles-for-kids /
https: // icebreakerideas.ru / загадки для детей /

200 Лучшие хитрые вопросы для детей, с ответами

Изображение: Shutterstock

Дети нуждаются в заботе по мере взросления. Вы можете вводить разные элементы, чтобы помочь им учиться и развиваться. Однако не все должно иметь отношение к ученым. Они тоже жаждут веселья. Задавать им каверзные вопросы может быть интересным занятием. Это улучшает их способность к латеральному мышлению, повышает их навыки решения проблем и сохраняет их вовлеченность в течение длительного времени.Кроме того, это поможет вам связаться с ними и улучшить их социальные навыки.

В этом посте мы предлагаем вам 200 вопросов-подсказок для детей, которые помогут улучшить их способности к творческому и критическому мышлению. Не раскрывайте ответы сразу; вместо этого побуждайте их думать и гадать. Мы уверены, что вы и ваши малыши сможете хорошо провести время в будущем.

200 вопросов с трюками для детей

1. Я дырявая, но могу выдержать критику. Кто я?

Губка

2. Что имеет много ключей, но не может открыть дверь?

Фортепиано

3.Что всегда приходит, но никогда не приходит?

Завтра

4. Я люблю танцевать, вертеться и прыгать, я трясу хвостом, как далеко плыву, бескрылый я летаю в небо. Что я?

Воздушный змей

5. Забери мое первое письмо, и я остаюсь прежним. Забери мое последнее письмо, и я остаюсь неизменным. Убери все мои письма, и я остаюсь собой. Что я?

Почтальон

6. У него один, а у человека два, у гражданина три, у человека четыре, у личности пять, а у жителя земли шесть.Что я?

Слог

7. Когда 1500 плюс 20 и 1600 минус 40 одно и то же?

Военное время

8. Мужчина замечает, что карманы его брюк пусты. Но кое-что в этом есть. Что бы это могло быть?

Отверстие

9. Что это такое, чем больше вы берете, тем больше становится?

Отверстие

10. Чем нельзя пользоваться, пока не сломается?

Кокос

11. Каков конец, к которому мы все хотим прийти?

Дивиденды

12.У синего дома голубые кирпичи; желтый дом из желтого кирпича. Из чего сделан теплый дом?

Стекло

13. 5 5 5 5 5 — Добавьте символ где-нибудь в этом ряду из пятерок, чтобы получилось уравнение, равное 500.

555-55 = 500

14. Какие платья нельзя носить никогда?

Адрес

15. Кто имеет один глаз, но не видит?

Игла

16. Если есть, то не делюсь. Если я поделюсь им, у меня этого не будет. Что это?

Секрет

17.В комнате пять сестер. Энн читает книгу, Маргарет готовит, Кейт играет в шахматы, Мари стирает белье. Чем занимается пятая сестра?

Играет в шахматы с Кейт.

18. Я там, где вчера следует сегодня, а завтра посередине. Что я?

Словарь

19. Вы всегда найдете меня в прошлом. Я могу быть создан в настоящем, Но будущее никогда не испортит меня. Что я?

История

20. Какая любимая игра Торнадо?

Twister

21.Что намокает при сушке?

Полотенце

22. Что представляет собой семибуквенное слово, содержащее тысячи букв?

Почтовый ящик

23. Что такого деликатного, что даже упоминание о нем ломает?

Silence

24. Какие две вещи нельзя есть перед завтраком?

Обед и ужин

25. Что можно сломать, но нельзя удерживать?

Обещание

26. Что живет, если его накормят, и умирает, если вы его напоите?

Пожар

27.Какое слово написано неправильно в каждом словаре?

Неправильно

28. Что никогда не задает вопросов, но всегда получает ответы?

Телефон

29. Что можно поймать, что не брошено?

Холодно

30. Что идет вверх, но никогда не падает?

Возраст

31. Что начинается с «е» и заканчивается на «е», но содержит только одну букву?

Конверт

32. Если самолет разбивается на границе США и Канады, где хоронят выживших?

Выживших не хоронят

33.Если восемь человек возводят стену за десять часов, сколько времени потребуется четырем мужчинам?

Ничего. Стена уже построена

34. Как можно без сна 25 дней?

Сон ночью.

35. Если у вас есть миска с шестью яблоками и вы берете четыре, сколько у вас есть?

Четверка, которую вы взяли

36. Как заставить исчезнуть номер один?

Добавьте букву «G» — и все!

37. Что больше Бога и зла больше диавола? Богатые люди этого хотят, а бедные это имеют.А если съесть, умрешь?

Ничего

38. Его можно взломать, это можно сделать, можно сказать, в него можно сыграть. Что это?

Шутка

39. В пятницу в город приехал ковбой. Он пробыл в городе три дня и уехал в пятницу. Как такое возможно?

Его лошадь зовут Пятница

40. Если зеленый человек живет в оранжерее, фиолетовый человек живет в фиолетовом доме, синий человек живет в синем доме, желтый человек живет в желтом доме, черный человек живет в черном доме.Кто живет в Белом доме?

Президент

41. У меня нечетное число. Убери одну букву и я стану ровным. Какой я номер?

Seven

42. Если вы поставите чашку на стол лицевой стороной на юг, находясь на северной стороне стола, с какой стороны находится ручка чашки?

За пределами

43. Почему учебник по математике грустный?

Потому что у него проблемы

44. У меня тысяча колес, но я не двигаюсь. Называй меня тем, кто я есть, звони мне почаще.

Парковка

45. Некоторые быстро ее забирают. Других нужно уговаривать. Те, кто выбирают это, выигрывают и теряют больше всего.

Риск

46. Что принадлежит вам, но используется другими людьми больше, чем вами?

Ваше имя

47. Какое английское слово сохраняет такое же произношение даже после того, как вы уберете четыре из пяти букв?

Очередь

48. Люди покупают меня, чтобы поесть, но никогда не едят. Что я?

Табличка

49.Что всегда ложится спать в обуви?

Лошадь

50. Прекрасная и круглая, Я сияю тусклым светом, Выросшая во тьме, женское наслаждение.

Жемчужина

51. Я могу быть тонкой, как рамка для картины, но у меня внутри есть много вещей, которые вы можете увидеть.

Телевидение

52. Что происходит раз в минуту, два раза в мгновение и никогда в тысячу лет?

Буква М

53. Какие два слова, вместе взятые, содержат больше всего букв?

Почтовое отделение

54.Если обезьяна, белка и птица мчатся на вершину кокосовой пальмы, кто первым получит банан?

На кокосовых пальмах не растут бананы.

55. У меня есть зубы, но я не могу есть. Что я?

Гребень

56. Если десять птиц сидят на дереве и охотник стреляет в одну, сколько птиц осталось на дереве?

Нет. Все птицы улетают

57. У меня семь букв и есть что-нибудь. Моя единственная анаграмма может помочь вашей боли. Если убрать мои первые две буквы, я изнашиваюсь.Удаление первых трех букв — это прилагательное, а удаление первых четырех букв оставляет некоторое время. Что я?

Колбаса

58. У чего четыре колеса и мухи?

Мусоровоз

59. Какая птица может поднять наибольший вес?

Кран

60. Что поднимается, когда идет дождь?

Зонт

61. Чем больше возьмешь, тем больше оставишь. Что я?

Следы

62.Если петух кладет яйцо на крышу сарая, в какую сторону оно катится?

Петухи не откладывают яйца

63. Как поднять слона одной рукой?

Нельзя, потому что у слона нет рук.

64. Что путешествует по миру, застряв на одном месте?

Штамп

65. Как можно уронить сырое яйцо на бетонный пол и не расколоть его?

Бетонный пол трудно взломать

66. Какой размер со слона, но при этом абсолютно ничего не весит?

Тень слона

67.Какое слово начинается с IS, заканчивается на AND, а в середине стоит LA?

Остров

68. Что движется быстрее: жара или холод?

Жара, потому что всегда можно простудиться.

69. Нападаю тяжело, а вот назад нет. Что я?

Тонна

70. У матери Бет три дочери. Одного зовут Лара, а другого — Сара. Как зовут третью дочь?

Beth

71. Если бы вы участвовали в гонке и обогнали человека, занявшего второе место, на каком месте вы были бы сейчас?

Второй

72.Какое дерево можно носить в руке?

Пальма

73. Что прыгает, когда ходит, и что сидит, когда стоит?

Кенгуру

74. Ребенок отца, ребенок матери, но ни один сын?

Дочь

75. Какой сын не хочет родитель?

Поджог

76. Я насекомое, и первая половина моего имени указывает на другое насекомое. У некоторых известных музыкантов было имя, похожее на мое. Что я?

Жуков

77.Тот, кто делает это, продает. Тот, кто покупает, этим не пользуется. Тот, кто его использует, не знает, что он им пользуется. Что это?

Гроб

78. Какой зонт большинство людей носит с собой в дождливый день?

Мокрая

79. Вы меня выгоняете, когда я вам нужен; ты вернешь меня, когда закончишь. Что я?

Якорь

80. Что будет, если бросить белую шляпу в Черное море?

Шляпа промокает

81. Отдать меня тому, кому я не принадлежу, — трусость, но взять меня — благородно.Я могу быть игрой, но никто не побеждает. Что я?

Виноват

82. У меня есть конец, но нет начала, дом, но нет семьи, пространство без комнаты. Я никогда не говорю, но я не могу сказать ни слова. Что я?

Клавиатура

83. Если я улыбаюсь, она тоже улыбается. Если я плачу, он тоже плачет. Если я кричу, ничего не происходит. Что это?

Зеркало

84. Он был женат на многих женщинах, но никогда не был женат. Кто он?

Священник

85. С острыми клыками сижу и жду, с пронзительной силой служу судьбе.Хватая бескровных жертв, провозглашая мою мощь; физическое соединение одним укусом. Что я?

Степлер

86. Что на вкус лучше, чем запах?

Язык

87. Я то, что многим людям, в том числе и вам, не нравится. Но меня вызывают каждый раз, когда кто-то ранен. У меня пять букв, и когда моя последняя буква ставится перед первой, я становлюсь страной. Что я?

Pains

88. Что такое, что после того, как вы уберете все, что-то еще останется?

Здоровый

89.У меня нежный голос, тонкая талия, меня часто приглашают поиграть. Но куда бы я ни пошел, я должен поклониться, иначе мне нечего сказать. Что я?

Скрипка

90. Я в меднике, но не в собаке. Я в процентах, но не в деньгах. Что я?

Letter C

91. Выстрелите в меня тысячу раз, и я все еще могу выжить; одна царапина от вас, и я обнаружу, что ваши перспективы резко упадут. Что я?

Шарик восьмерки

92. Сдерживающая рука.Он удерживает нас от ужасных поступков, и с ним тяжело жить. Что это?

Вина

93. Сначала я один, потом я не кажусь никем, в смерти я рожу новую жизнь. То, что поднято, превосходит меня, потому что, стоя на коленях, я добавляю к миру, который изобилует. Что я?

Семя

94. Утомляет лошадь, беспокоит человека. Отгадай мне эту загадку, если сможешь.

Седло

95. Что бегает по двору без движения?

Забор

96. Что произойдет, если бросить синий камень в желтое море?

Тонет

97.Четыре ноги, зубчатые зубы. Флот движения, воды и земли. У меня нет настроения; для меня ты еда, пока я тащу тебя под землю.

Аллигатор

98. Где 11 + 2 = 1?

В часах

99. Я то, что видно только в темноте. Быстрее всех и почти так же стары, как время; Дальний брат Дэй; огонь и обморок, я без тени зажигаю — сможете ли вы разгадать эту рифму?

Звездный свет

100. Что ты не видишь, что всегда перед тобой?

Будущее

101.Что такое золото, когда оно старое, и серебро, когда оно новое, которое трудно найти, но легко потерять, стоит дорого, но бесплатно?

Друг

102. За свою жизнь я умираю дважды: один раз завернутый в шелк, один раз покрытый пылью.

Гусеница

103. Какой популярный сыр готовят в обратном направлении?

Эдам

104. Как вы можете физически стоять за своим другом, если он физически стоит за вами?

Спина к спине

105. В них часто оказываются плохо воспитанные дети.

Уголки

106. Я закрываю то, что реально, и скрываю то, что правда. Но иногда я пробуждаю в тебе храбрость. Что я?

Грим

107. Какой бант нельзя завязать?

Rainbow

108. Дом с двумя жителями, иногда с одним, редко с тремя. Сломай стены, съешь границы, а потом выбрось меня. Что я?

Арахис

109. У меня триста голов крупного рогатого скота с одним носовым шнуром.

Бусы

110. Я начало печали и конец болезни.Вы не можете выразить счастье без меня, а я среди крестов. Я всегда в опасности, но никогда не в опасности. Вы можете найти меня на солнце, но я никогда не выхожу из темноты.

Буква S

111. Естественное состояние, меня все ищут. Иди со мной, и ты упадешь. Вы делаете меня, когда тратите, и вы используете меня, когда едите без конца. Что я?

Баланс

112. Я довольно крупный и обычно величественный. Я всех оттенков радуги. Я могу тебя съесть, я могу тебя нагреть. Вы только хотели бы видеть меня.Что я?

Дракон

113. Медленно вытягивая руки, я поднимаюсь и двигаюсь к теплу. Яркие цвета, мои сестры и я. Кто мы?

Цветы

114. Хотя я далек от сути, я не ошибся. Я твое исправлю. Что я?

Ластик

115. На каком корабле нет капитана, кроме двух помощников?

Ухаживание

116. Руки у нее есть, но не держатся, зубы есть, но не кусаются, ноги у нее холодные, глаза у нее незрячие.Кто она?

Кукла

117. Мы изумруды и бриллианты, потерянные луной, найденные солнцем и скоро найденные.

Dew

118. Меня нельзя пощупать, увидеть или потрогать; тет Меня можно найти в каждом. Мое существование всегда вызывает споры, но есть музыкальный стиль, названный в честь меня.

Душа

119. Что покупает двор, что носит на ноге?

Ковер

120. Он маленький, но может взобраться на башню.

Муравей

121.Сила десяти мужчин, длина десяти мужчин, десять мужчин не сломают ее, но мальчик уходит с ней. Что я?

Веревка

122.
Что прыгает, когда идет, и что сидит, когда стоит?

Letter V

123. Я всего лишь дыры, привязанные к дыркам, но я крепок как железо.

Цепь

124. Я много работаю почти каждый день, мало времени на танцы и игры. Если бы я мог достичь того, чего хочу, все, как я, сейчас ушли бы на пенсию. Что я?

Врач

125.Удар за ударом, они подходили друг другу. Ни один не упадет на другой. В глазах толпы они были такими.

Равно

126. Какие улицы нравятся привидениям?

Deadend

127. Глубокий колодец, полный ножей.

Рот

128. Вы можете получить это и быть на этом, но это никогда не будет длиться вечно.

Peace

129. Я прожил много лет и до сих пор чувствую себя молодым. Я пережил депрессии, спады и даже тысячелетия. Я богаче самого богатого из мужчин.Вы можете навещать меня, но не моих владельцев. Меня показали по телевизору, и я могу взять и отдать вам то, что принадлежит вам, но только если вы попросите меня. Так скажи мне, кто или что я?

Банк

130. Какое слово становится короче при добавлении к нему двух букв?

Короткий

131. У г-жи Браун пять дочерей. У каждой из этих дочерей есть брат. Сколько детей у миссис Браун?

Шесть

132. Вы можете получить его только после того, как отдадите его.

Респект

133.Многие слышали меня, но никто меня не видел, и я не буду отвечать, пока с ними не поговорим. Что я?

Echo

134. Хотя моя корова мертва, я все равно ее бил. Какой рэкет она делает!

Барабан

135. Поехал в лес и достал, когда достал. Не хотел, искал, не нашел, взял домой.

Осколок

136. Чем всегда все кончается?

Letter G

137. Что падает, но никогда не ломается и не ломается, но никогда не падает?

Ночь и день

138.Я такой простой, что только указываю, но при этом веду людей по всему миру. Что я?

Компас

139. Безрукий, безногий, в юности ползаю. Тогда придет время сменить сон. Я проснусь, как новорожденный, летающий зверь, — а пока пирую останками мертвых.

Личинки

140. Сижу в парламенте. Ты увидишь меня только ночью. Что я?

Сова

141. В каком слове из пяти букв осталось одно, если убрать две?

Камень

142.Шаг мой медленный, снег — мое дыхание. Растираю землю. Мой поход положил конец тому, что я был убит солнцем или утонул в море.

Ледник

143. Я один из многих, вы забываете, что я здесь, но я просто внизу без меня, вы наверняка рухнете. Давай, что я?

Пальцы

144. Хожу сухой, а выхожу мокрым; чем дольше я нахожусь, тем сильнее становлюсь. Что я?

Чайный пакетик

145. В чем разница между хорошо одетым мужчиной на велосипеде и плохо одетым мужчиной на трехколесном велосипеде?

Шина

146.Я рожден в страхе, воспитан в истине и на деле прихожу к своему собственному. Когда приходит время, когда меня вызывают, я прихожу служить делу нужды.

Мужество

147. Что повсюду в доме?

Крыша

148. Какие два числа совпадают при умножении или сложении их?

2 x 2 = 4, 2 + 2 = 4

149. Я видел человека в белом; он выглядел весьма привлекательно. Он не был стар, но стоял на холоде. И когда он почувствовал солнце, он побежал.Кем он мог быть? Пожалуйста, ответь мне.

Снеговик

150. Мой первый в руках, рубит кости и костный мозг. Второй — в клинке, выкованном из холодной стали. Моя третья по арбалету, а также по стрелам. У меня четвертый в силе, пробитый щитом. Моя пятая — в чести, а также в клятвах. Мой последний конец всему этому.

Оружие

151. Не рожденный, но извлеченный из тела Матери, я висю, пока половина меня не исчезнет. Я сплю в пещере, пока не состарюсь, а потом ценится за мое закаленное золото.Что я?

Сыр

152. Капитан принял ванну, не промокнув живот.

Каноэ

153. В какой комнате нет физических стен?

Чат

154. У меня нет жизни, но я могу умереть, кто я?

Аккумулятор

155. Апрельские ливни приносят майские цветы. Что приносят майские цветы?

Паломники

156. Разбейте его и лучше, сразу поставьте, а сломать тяжелее снова.

Запись

157.Если у тебя есть я, ты хочешь мне сказать. Если ты мне скажешь, меня у тебя нет.

Тайна

158. Рожденный в печали, возрастающий с возрастом, нужно много, чтобы быть мудрецом. Что это?

Мудрость

159. Для меня есть два значения. С одним мне, возможно, придется сломаться. С другой я держусь. Моя любимая характеристика — моя очаровательная ямочка. Что я?

Галстук

160. Ты легко можешь дотронуться до меня, но не увидеть меня. Вы можете выбросить меня, но не прочь. Что я?

Ваша спина

161.Если вы живете в 11-этажном доме и все зеленое, дом, блендер, туалет, лифт, ленточка, диван, компьютер, тарелки, еда! Вы попадаете на 6-й этаж, а лестниц больше нет. Как вы попали на 11 этаж?

Зеленый лифт

162. Чтобы разгадать меня, тебе нужен ключ. Не ключ, сделанный рукой слесаря, но ключ, который пойму только я. Что я?

Шифровальный ключ

163. У меня есть ветви, но нет фруктов, ствола или листьев. Что я?

Банк

164.Два в углу, один в комнате, ноль в доме, но один в убежище. Что это?

Буква «R»

165. Чем больше этого, тем меньше видишь. Что это?

Darkness

166. Что такое черное, когда оно чистое, и белое, когда оно грязное?

Классная доска

167. Я легок как перышко, но самый сильный человек не может удержать меня и пять минут. Что я?

Дыхание

168. Меня находят в носках, шарфах и рукавицах; и часто в лапках игривых котят.Что я?

Пряжа

169. У кого много глаз, но чего не видно?

Картошка

170. У кого есть руки, но нет возможности хлопать в ладоши?

Часы

171. Кто имеет ноги, но не ходит?

Стол

172. Какая группа никогда не играет музыку?

Резинка

173. Что имеет слова, но не говорит?

Книга

174. Что имеет большой палец и четыре пальца, но не рука?

Перчатка

175.У чего есть голова и хвост, но нет тела?

Монета

176. В каком здании больше всего этажей?

Библиотека

177. У чего 13 сердец, но нет других органов?

Колода карт

178. Я начало всего, всему конец. Я начало вечности, конец времени и пространства. Что я?

Буква «Е»

179. Он бродит по сельской местности ушами, которые не слышат. Что это?

Кукуруза

180.Какое пальто лучше всего надевать мокрым?

Слой краски

181. Если двое составляют компанию, а трое — толпа, то какие четыре и пять?

Девять

182. Какие три числа, ни одно из которых не равно нулю, дают одинаковый результат независимо от того, сложены они или умножены?

Один, два и три

183. Три доктора сказали, что Билл был их братом. Билл говорит, что у него нет братьев. Сколько братьев на самом деле у Билла?

Нет. У него есть три сестры.

184.В машине два отца и два сына, а в машине всего три человека. Как?

Это дед, отец и сын.

185. Позавчера мне был 21 год, а в следующем году — 24. Когда у меня день рождения?

31 декабря; сегодня 1 января.

186. Мужчина описывает своих дочерей, говоря: «Все они блондинки, кроме двух; все брюнетки, кроме двух; и все рыжеволосые, кроме двоих. Сколько у него дочерей?

Трое: блондинка, брюнетка и рыжая

187.Если есть три яблока, и вы убираете два, сколько у вас яблок?

У вас есть два яблока

188. Слово, которое я знаю, шесть букв в нем, удалите одну букву и осталось 12. Что это?

Десятки

189. Вы видите меня один раз в июне, два раза в ноябре и совсем не в мае. Что я?

Буква «E»

190. Какое четырехбуквенное слово можно писать вперед, назад или вверх ногами и при этом читать слева направо?

Полдень

191.Что такое 3/7 курицы, 2/3 кошки и 2/4 козы?

Чикаго

192. Я слово из трех букв; добавить два и меньше будет. Какое я слово?

Немного

193. Я — слово, начинающееся с буквы «Я». Если вы добавите ко мне букву «а», я стану новым словом с другим значением, но оно будет звучать точно так же. Какое я слово?

Isle

194. Какое слово в английском языке означает следующее: первые две буквы означают мужчину, первые три буквы означают женщину, первые четыре буквы означают великое, а весь мир означает великую женщину.Что за слово?

Героиня

195. Кто может бегать, но никогда не ходит, имеет рот, но никогда не разговаривает, имеет голову, но никогда не плачет, имеет кровать, но никогда не спит?

Река

196. Что может заполнить комнату, но не занимает места?

Свет

197. Люди заставляют меня, спасают меня, меняют, воспитывают. Что я?

Деньги

198. Что проходит через города и поля, но не движется?

Дорога

199. У меня озера без воды, горы без камня и города без построек.Что я?

Карта

200. Что человек любит больше жизни, что ненавидит больше, чем смерть или смертельные раздоры, то, чего желают довольные люди; бедные имеют, богатые нуждаются; скупец тратит, расточитель копит, а всех несут в могилу?

Ничего

Загадки надолго развлекают детей, заставляют думать, рассуждать, задавать вопросы и учиться. Кроме того, они делают забавное занятие в скучное время или на вечеринках по случаю дня рождения. Это отличный способ провести с детьми время, не связанное с медиа или технологиями.Так что будьте готовы задать своим детям эти наводящие на размышления вопросы и хорошенько поупражняйте их мозг, и, прежде чем вы это узнаете, они будут собирать свои собственные загадки, чтобы поделиться с семьей!

Рекомендованные статьи

Сколько вы можете правильно ответить?

Что касается хитрых вопросов, то чем больше смотришь, тем меньше видишь.

После того, как вы узнаете ответы на хитрый вопрос, вы начинаете спрашивать себя, как вы могли пропустить что-то такое простое.

По правде говоря, большинство вопросов-уловок созданы для того, чтобы обмануть ваш разум, поэтому ответы на вопросы-уловки обычно нелогичны и забавны.

Теперь позвольте мне спросить вас, вы верите, что вас нельзя обмануть с помощью таких вопросов?

Как вы думаете, сможете ли вы быстро найти ответы даже на самые сложные вопросы с подвохом?

Что ж, давай, проверь свои способности с помощью вопросов с хитростями, перечисленных ниже.

Список вопросов и ответов с трюками:

Вот список сложных вопросов-трюков и ответов, на которые вы можете задать любой или ответить сами:

  1. Что такое, что ломается, но не падает, а что падает и не может сломаться?

Показать ответ

День прерывается, затем наступает ночь.

  1. У вас есть единственная спичка, и вы находитесь в черной как смоль комнате со свечой, масляной лампой и газовой плитой. Что вы зажигаете первым?
  1. Если у фермера было 322 коровы, и он сделал один выстрел, и все коровы погибли. Как это удалось фермеру?

Показать ответ

Фермер сделал панорамный снимок.

  1. Какая-то семья жила в сарае. Однажды, когда отец вернулся с работы, он нашел свою жену мертвой.Один из детей сказал, что смотрел телевизор, второй сказал, что рисовал, а последний ребенок сказал, что читал из-за угла. Кто убил мать?

Показать ответ

Последний с момента рождения дом круглый и без углов.

  1. Из букв алфавита, что делает мед?
  1. Можете заполнить пустое место; 19 = 8, 6 = 3, 70 = 7, 8 = 5, 4 = 4, 60 = 5, 15 = 7, 16 =? 17 =?

Показать ответ

16 = 7 и 17 = 9, потому что количество букв в написании шестнадцать равно 7, а семнадцать — 9.

  1. Не могли бы вы перечислить числа, в написании которых есть буква А, от 1 до 100?
  1. Какой правильный? «Пингвины летят» или «Пингвин летят».

Показать ответ

Нет, потому что пингвины не летают.

  1. Мой друг, Джон — 45-летний кузнец ростом 7 футов, который постоянно ест. Можете ли вы определить, что Джон ест больше всего?
  1. Если бы вы спрыгнули с крыши трехэтажного дома, где бы вы приземлились?
  1. Что это за вещь, которую, если вы дадите кому-то, человек, скорее всего, немедленно вернет ее?
  1. Кто станет президентом в случае смерти вице-президента?

Показать ответ

Действующий президент.

  1. Если обезьяна, птица и белка забегают на кокосовую пальму. Какой из них, скорее всего, достигнет банана?

Показать ответ

Ни одного из них. На кокосовых пальмах не растут бананы.

  1. Если у птицы 7 яиц и динозавры съели одно из ее яиц. Сколько яиц осталось бы у птицы?

Показать ответ

Динозавры вымерли, поэтому яйца есть не будут.

  1. Я что-то, люди очень много работают, чтобы заполучить меня каждый день.Когда они меня поймают, они меня отдают. Что я?
  1. Что идет вверх, но никогда не падает?
  1. Сколько у вас пальцев?

Показать ответ

8, так как ваши большие пальцы не в счет.

  1. Если на дереве 10 птиц и охотник стреляет в 1 птицу. Сколько птиц останется?

Показать ответ

Нет, потому что остальные птицы испугаются и улетят.

  1. Как нарисовать квадрат из трех линий?

Показать ответ

Вы рисуете квадрат, а затем рисуете три линии в центре квадрата.

  1. Что становится влажнее и влажнее по мере высыхания?
  1. У кого четыре ноги, но не может ходить?
  1. Почему эскимосы не могут есть яйцо пингвина?

Показать ответ

Потому что они живут на противоположном конце света!

  1. Почему женщина взяла слона вместо машины?

Показать ответ

Из-за большего ствола.

  1. Как заставить исчезнуть номер один?

Показать ответ

Добавьте букву G, и она исчезнет.

  1. Что такое черно-белое, что читается?

Показать ответ

Газеты — вы читаете их полностью.

  1. Почему лошади не носят перчатки?

Показать ответ

У них нет рук.

  1. Сколько раз можно отнять 2 из 100?

Показать ответ

Один раз.После принятия первых двух у вас больше не будет 100.

  1. Что мышь сказала сыру?
  1. Какая гора была самой высокой до открытия Эвереста?

Показать ответ

Гора Эверест все еще существовала, просто она не была открыта.

  1. Что начинается с «е», заканчивается на «е» и состоит только из одной буквы?
  1. Если у парня 320 яиц, а вы взяли 37 яиц, сколько яиц у вас есть?

Покажите ответ

37, который вы взяли.

  1. Что тяжелее: тонна металла или тонна пера?

Показать ответ

Оба одинаковы.

  1. Какие два дня недели начинаются с буквы Т, кроме вторника и четверга?

Показать ответ

Сегодня и завтра.

  1. Назовите то, что нельзя есть во время завтрака?
  1. Когда петух откладывает яйца на наклонной крыше, в какую сторону он катится?

Показать ответ

Петухи не откладывают яйца.

  1. Как водитель автобуса увидел собаку на дороге во время отключения электроэнергии?

Показать ответ

Это было днем.

  1. Что общего между Генрихом Восьмым и Кермитом?

Показать ответ

То же отчество.

  1. Какое слово из 10 букв содержит тысячи слов?

Показать ответ

Словарь.

  1. Что нормально встает и опускается?
  1. Что становится со снегом, когда он тает?
  1. Почему мы пьем воду?

Показать ответ

Потому что воду нельзя жевать.

  1. Какая следующая буква после N в этом ряду? JFMAMJJASON. . .

Показать ответ

D — На декабрь.

  1. Почему пингвины не живут в Англии?

Показать ответ

Они боятся Уэльса.

  1. Как вы называете людей, которые приехали из страны медалей?
  1. Почему Золушку выгнали из футбольной команды?

Показать ответ

Она убежала от мяча.

  1. Если электричка едет на юг, в каком направлении движется дым?

Показать ответ

Электропоезда не курят.

  1. Подумайте, что будет, если бросить зеленый камень в Красное море?
  1. Когда 1 + 1 не равно 2?

Показать ответ

Когда вы сделали ошибку.

  1. В каком семибуквенном слове сотни букв?
  1. Можно ли человеку не спать 8 дней?

Показать ответ

Он будет спать по ночам.

  1. Вы не можете назвать меня по имени, не сломав меня? Что я?
  1. Что произойдет, если смешать pay и pear?
  1. Что произойдет, если смешать с и солнце?
  1. Какие три буквы превращают девушку в женщину?
  1. Что увеличивается, но никогда не уменьшается?
  1. Что имеет голову ночью, а днем ​​ее нет?
  1. Чем больше вы берете из него, тем больше он становится.Что это?
  1. Когда пятница всегда раньше четверга?

Показать ответ

В словаре.

  1. Какое дерево можно носить в руке?
  1. Что такое красное и белое и может путешествовать по миру за одну ночь?
  1. Где была подписана декларация независимости?

Показать ответ

Внизу.

  1. Что идет вверх и вниз и остается на одном месте?
  1. Если мы с тобой тупые, кто между нами?
  1. Что стена сказала другой стене?

Показать ответ

Встретимся на углу.

  1. Что происходит раз в минуту, два раза в мгновение и никогда в тысячу лет?
  1. Какая змея самая ядовитая в мире?

Показать ответ

Никаких ядовитых змей, только ядовитые.

  1. Что математики ели на ужин?
  1. Что такое зеленый и красный и идет со скоростью 90 миль в час?

Показать ответ

Лягушка в блендере.

  1. Как сложить восемь шестерок, чтобы в сумме получилось 750?

Показать ответ

666 + 66 + 6 + 6 + 6 = 750.

А теперь честно говоря, сколько из этих вопросов обмануло вас? Получилось ли у вас правильно с первой попытки? Также, если есть ответ, которого вы не понимаете, оставьте комментарий, и я вам объясню.

Не забудьте попробовать эти довольно сложные вопросы со своими друзьями и близкими.

50 сложных загадок и головоломок для детей с ответами

Последнее обновление

Один из лучших способов стимулировать юные умы — вовлечь их в мозговой штурм, а решение загадок — лучший способ улучшить мыслительные способности маленьких детей. Дети не только получают массу удовольствия от разгадывания загадок, но и осваивают различные новые навыки.Вот несколько интересных и креативных загадок или головоломок для детей, которые улучшат их когнитивное развитие.

Как загадки помогают детям?

Даже такая простая вещь, как загадка, может творить с детьми чудеса множеством способов. Вот некоторые из преимуществ загадок для детей.

1. Загадки ликвидируют разрыв между родителями и детьми и укрепляют связи. Это также помогает детям преодолевать социальные проблемы, когда они взаимодействуют с другими и передают загадки.

2. Они поощряют решение проблем и критическое мышление в молодом возрасте, что является жизненно важными навыками. Это заставляет их мыслить творчески и работать мозгами.

3. Загадки полезны для улучшения словарного запаса детей, когда им бросают слова, которые они не понимают, и они пытаются понять, что они означают.

4. Понимание является важной частью обучения в школе. Дети могут понимать слова или фразы из контекста и определять детали, которые в противном случае были бы проигнорированы.

5. Смех и юмор жизненно важны, а загадки часто добавляют умственного юмора. Они расслабляют умы детей, обучая их так, как им интересно.

Также читайте: 20 простых и популярных скороговорок для детей

50 простых загадок для мозга с ответами для детей

От простых загадок для детей до сложных загадок, мы будем обсуждать различные типы загадок, которые помогут отточить их навыки мышления. Мы также будем обсуждать загадки для детей, чтобы получить больше удовольствия от общения с семьей и друзьями.Помимо загадок, вы также можете попробовать другие развлечения и игры, которые будут интересными и познавательными. Ниже приведены несколько безумных, забавных и умных английских загадок для детей, которые сделают обучение более увлекательным:

1. Загадка: Что начинается с буквы «т», заполнено буквой «т» и заканчивается буквой «т»?

Ответ: Чайник с чаем.

2. Загадка: Какой самый большой английский алфавит содержит больше всего воды?

Ответ: Буква «С».

  1. Загадка: Сможете угадать, что находится в конце радуги?

Ответ: Буква «W».

  1. Загадка: Что будет весить больше: фунт хлопка или фунт железа?

Ответ: Ни один из них не весит больше или меньше, поскольку они весят один фунт одинаково.

  1. Загадка: В одноэтажном доме красное кресло, красная кровать, красный компьютер, красные цветы, красный стол, красная ковровая дорожка — все вокруг красного цвета.Какого цвета лестница?

Ответ: Дом одноэтажный, следовательно, лестницы нет.

  1. Загадка: У меня есть лицо и две руки, но нет ни рук, ни ног. Что я?

Ответ: Часы.

  1. Загадка: Что начинается и заканчивается буквой «Е», но имеет только одну букву?

Ответ: Конверт.

  1. Загадка: Почему нельзя похоронить человека, живущего в Нью-Йорке, в Чикаго?

Ответ: Потому что он жив.

  1. Загадка: Что нужно сломать перед использованием?

Ответ: Яйцо.

  1. Загадка: В каком месяце года 28 дней?

Ответ: Во всех месяцах 28 дней.

11. Загадка : Где ты найдешь пятницу до четверга?

Ответ: В словаре.

  1. Загадка: Томми бросает мяч так сильно, как только мог, и тот возвращается к нему, при этом никто и никто его не касается.Как?

Ответ: Подбросил мяч вверх.

  1. Загадка: У чего есть шея, но нет головы?

Ответ: Бутылка.

  1. Загадка: Что один океан сказал другому океану?

Ответ: Ничего не сказал. Он просто помахал.

  1. Загадка: В чем дыры повсюду, но все еще сдерживается?

Ответ: Губка.

  1. Загадка: Девушка упала с 40-футовой лестницы, но все равно не пострадала. Почему?

Ответ: Она упала с нижней ступеньки.

  1. Загадка: Что часто используется другими, но принадлежит вам?

Ответ: Ваше имя.

  1. Загадка: Что поднимается, но никогда не опускается?

Ответ: Возраст.

  1. Загадка: На что всегда отвечают, не задавая вопросов?

Ответ: Дверной звонок.

  1. Загадка: Представьте, что вы находитесь посреди моря. В вашей лодке есть дыра, и вы окружены акулами. Что ты будешь делать?

Ответ: Хватит воображать.

21. Загадка: Какие два ключа не могут открыть никакие двери?

Ответ: Осел и обезьяна.

  1. Загадка: Сможете ли вы угадать самый простой способ удвоить свои деньги?

Ответ: Поставьте перед зеркалом.

  1. Загадка: Загадка есть у каждого человека, и никто не может ее потерять. Сможете угадать, что это?

Ответ: Тень.

  1. Загадка: Какое слово написано неправильно в каждом словаре?

Ответ: Слово «Неправильно».

  1. Загадка: Какое дерево можно носить в руке?

Ответ: Ладонь

  1. Загадка: Может ли человек десять дней не спать?

Ответ: Это возможно, потому что все мы спим всю ночь, а не днем.

  1. Загадка: Он заполнен ключами, но не может открыть ни одну дверь. Что это?

Ответ: Фортепиано.

  1. Загадка: Что есть рог, но не шумит?

Ответ: Носорог.

  1. Загадка: Что размером со слона, но ничего не весит?

Ответ: Тень слона.

  1. Загадка: На чем надета обувь даже во время сна?

Ответ: Лошадь.

31. Загадка: Если бросить белый камень в Красное море, что с ним будет?

Ответ: Намокнет.

  1. Загадка: Вы можете угадать, что становится все влажнее и влажнее по мере высыхания?

Ответ: Полотенце.

  1. Загадка: Кто четвероногий, но не может ходить?

Ответ: Стол.

  1. Загадка: Птица, обезьяна, кошка и белка забрались на кокосовую пальму.Кто первым съест манго?

Ответ: Нет, так как они на кокосовой пальме и манго там нет.

  1. Загадка: Мои карманы пусты, но в них еще что-то есть. Что это?

Ответ: У них дырки.

  1. Загадка: Целыми днями летаю, но никуда не езжу. Что я?

Ответ: Я флаг.

  1. Загадка: Подавать можно, но нельзя есть?

Ответ: Теннисный мяч.

  1. Загадка: Его бьют и бьют, но он никогда не плачет. Что это?

Ответ: Яйцо.

  1. Загадка: У него четыре глаза, но он по-прежнему ничего не видит. Что это?

Ответ: Миссисипи.

  1. Загадка: Меня купили для еды, но люди меня не едят. Почему?

Ответ: Потому что я тарелка.

41. Загадка: Я английское слово.Если вы произносите меня правильно, то вы ошибаетесь, а если вы произносите меня неправильно, то вы правы. Какое я слово?

Ответ: Слово «Неправильно».

  1. Загадка: Мой плюшевый мишка никогда не бывает голоден. Вы можете догадаться, почему?

Ответ: Потому что он набит.

  1. Загадка: Как бы вы назвали человека, у которого не все пальцы в одной руке?

Ответ: Нормальный мужчина, потому что у вас пальцы на обеих руках.

  1. Загадка: Почему маленькая девочка закопала свой факел?

Ответ: Потому что у него сели батарейки.

  1. Загадка: Он начинается с «P», заканчивается на «E» и состоит из тысяч букв. Что это?

Ответ: Почтовое отделение.

  1. Загадка: Трое мужчин прыгают в воду, но только двое выходят с мокрыми волосами. Почему?

Ответ: Третий мужчина был лысым.

  1. Загадка: У отца Шерри пятеро дочерей — Саса, Сесе, Сиси, Сосо. Угадай имя пятой дочери.

Ответ: Если вы думаете, что это Сусу, то вы ошибаетесь. Это Шерри.

  1. Загадка: Что происходит раз в минуту, два раза в мгновение, но никогда за тысячу лет?

Ответ: Это буква «М».

  1. Загадка: Если вы едете на юг на электричке, куда идет дым от поезда?

Ответ: Нигде.Электропоезд не дымит.

  1. Загадка: Они приходят ночью без вызова и исчезают днем, не будучи украденными. Вы можете догадаться, что это такое?

Ответы: звёзд.

Загадки и головоломки не только прививают детям интеллектуальный юмор, но и стимулируют развитие мозга.

Comments